Sunteți pe pagina 1din 82

Procedural Skills

Sub-Unit of the Medical Skills Program

MDCN 320 & 420

UNDERGRADUATE MEDICAL EDUCATION


Core Document
Classes of 2017 & 2018
2015-2016 Academic Year
2015
Procedural Skills

CONTENTS PAGE

The Procedural Skills Course


Introduction I-3
Overview I-3
Objectives I-4
Evaluation I-4
Module Structure I-4
Schedules I-5

Modules
Module #1 - Suturing I-6
Module #2 - IV and Intraosseous Access I - 18
Module #3 - Lumbar Puncture I - 41
Module #4 - Basic Airway I - 29
Module #5 - Advanced Airway I - 45
Module #6 Casting & Splinting I - 57
Module #7 Ultrasound Skills and Paracentesis I - 66

Module #8 - OR Scrub (to be handed out separately)

I-2
UNIT CO-CHAIRS
Dr. Ian Wishart Dr. Anthony Chad
ianwishart@shaw.ca afchad@shaw.ca

Introduction
Welcome to the Procedural Skills Program, the newest component of the Medical Skills
Course. In years past, students and faculty have expressed dissatisfaction with the level of
procedural skills competency achieved by graduating medical students. Survey data from
the U of C showed that large portions of the graduating class were leaving medical school
without ever having performed such basic procedures as lumbar punctures, starting IVs,
intubation and splinting.
As a result of this, we have created a formal curriculum designed to aid in the development
of these procedural skills. Using a variety of simulation models, you will be taught the skills
necessary to perform these essential procedures with confidence in the real world. It is also
our goal that you will development the necessary knowledge base to know when these
procedures are indicated or contraindicated as well as possible complications of each
procedure.
Program Overview
The program currently consists of 6 individual modules, and a surgical scrub course. They
are:
Intravenous Placement / Intraosseous
Basic Suturing
Lumbar Puncture
Airway Management
Cast & Splint
Ultrasound
Surgical Scrub
Each module consists of two hours of direct teaching time, plus preparation time on your
part. It is important that the preparatory work be done in order to maximize the yield gained
from spending time with your preceptors and simulators. Attendance at these sessions is
mandatory for successful completion of the course.

The modules will be spread out over the year, as available curriculum time allows. Every
attempt is made to match the skill being taught to the systems course being taught at the
time, although this is not always possible.
In an effort to ensure that opportunities to solidify skill acquisition are available, students will
be given the opportunity to attend an optional practice session towards the end of each
academic year to practice the skills learned with preceptors available on an as-needed
basis. This will be purely optional.

I-3
Objectives
At the end of the Program, the student will:
Be able to explain and demonstrate the steps involved in each procedure
Competently perform of each skill in a simulation setting at the time of small group
teaching.
Be able to identify the indications for each procedure, as well as the relative and
absolute contra-indications.
Be able to describe the common and serious complications of each procedure as
well as strategies to minimize their occurrence and manage them when they occur.

Evaluation
Evaluation will take place as part of the Medical Skills 1 and Medical Skills 2 OSCE
Examinations. One station on each exam will be dedicated to procedural skills and will
require the student to perform the procedure in question using proper technique and may
involve some questions from the examiner regarding the procedure.

The first year exam station will, obviously, examine a skill taught during the first year. The
second year exam station may test any skill taught over both years of the program.

The procedural skills component of the Medical Skills OSCE is a certifying exam for both
first and second year students. Both exams will constitute 100% of the Procedural Skills
mark for each year.

Module Structure
Each module will function in essentially the same fashion, with the exception of the Surgical
Scrub Course, discussed below. Each module consists of 4 components:
A written document which describes the procedure being taught
An on-line video demonstrating the procedure
A brief pre-test which is reviewed at the start of the small group session. Please
bring your completed quiz with you and submit it to your preceptor
A two-hour small group practice session with an appropriate simulation, preceptored
by a faculty member with expertise in the skill being taught.

The preparatory materials (the written module, the video and the pretest) can all be
accessed through the Osler System. The videos can be accesses through the Year 1 &
year 2 podcast blogs and through the Core site on Osler.
The small group sessions will take place in either G820 or the ATSSL Lab and will consist
of 8 simultaneous groups of 8-10 students. Each session will only last two hours, so
although a procedural skills session is listed in the master timetable as lasting 4 hours,
each individual student is only committed to a two hour time period.
During the surgical scrub course, students are taught and given the opportunity to practice
the basics of functioning in the sterile environment of the operating room. These sessions
are kindly run by our surgical nursing colleagues. This module, unlike the others, includes
a large group didactic session and small groups which meet at various times. The reason
for this is that the sessions require an unoccupied OR and are staff intensive. Each group
will be scheduled individually for these sessions.

I-4
Program for Class of 2018 (First Year)
Module Date Page#
#1 - Suturing November 6th, 2015 I- 6
#2 - IV Placement / Intraosseous December 4th, 2015 I - 18
#3 - Lumbar Puncture February 26th, 2016 I - 41
#4 & 5 - Airway (Basic & Advanced) January 22nd, 2016 I - 29

OR Scrub/Sterile Techniques (Handouts to be Provided Separately)


Lecture: October 2 , 2016
nd

Practical: Via Sign-up process

Program for Class of 2017 (Second Year)


Module Date Page#
#6 Casting and Splinting September 24th, 2015 I - 57
#7 Ultrasound Skills & Paracentesis October 23rd, 2015 I - 66

I-5
Procedural Skills - Module #1

Suturing

I. Introduction
Welcome to the Procedural Skills Suturing Module. This module is comprised of this article, an online teaching
module and a hands-on skills session. This article is intended to provide you with the concepts needed to
understand suturing. It is to be used along with the online demonstration, available on Osler. The module will
then be followed by a hands-on skill session at a later date. Further practice sessions will also be available at
later dates.

II. Objectives
No teaching would be complete without a series of objectives, so here they are! By the end of this module the
student will:

(a) Be able to decide whether a wound is appropriate for suturing.


(b) Be able to provide anesthesia via local wound infiltration.
(c) Demonstrate safe-handling techniques for sharps involved in the procedure.
(d) Be able to don sterile gloves.
(e) Demonstrate the sterile preparation of a wound.
(f) Be able to demonstrate the correct technique for placing interrupted sutures.
(g) Demonstrate a proper instrument tie.
(h) Be able to discuss the indications for tetanus prophylaxis.

III. Principals of Wound Care


To Suture or Not To Suture? That is the question!
Suturing involves the closure of a wound, and the first step in this procedure is deciding whether suturing is
appropriate. Although we would like to close all wounds, sometimes this increases the risk of infection.
Unfortunately, there is no hard and fast rule as to when that risk becomes too high. A good generalization is
suturing a wound that has been open for more than 8 hours raises its risk of infection.

In reality, though, deciding to close a wound depends on (a) the tissue itself, (b) the length of time the wound
has been open (c) the degree of contamination of the wound and (d) the cosmetic or functional effect of the
wound.

The tissues around the wound play an obvious role in the healing process. Healthy tissue with good blood
supply heals well and extends the length of time one can wait to safely perform closure. For such reasons,
wounds of the scalp (with a large blood supply) heal marvelously, even with long delays to closure. But poorly
vascularized areas (i.e. the digits) often heal poorly. As we are trying to avoid infection, we must consider the
infection risk for each wound. These risks for infection can be divided into endogenous and exogenous risks.

Endogenous risks include any condition native to the tissue that sets it up for poor healing. This includes
underlying vascular conditions (vasculitis, peripheral vascular disease, etc.), immunosuppression (HIV,
immunosuppressive drugs, etc.), or areas of usually low vascularity (i.e. the distal extremities).

I-6
Exogenous factors that play a role in the blood supply include devascularization of the tissue by the wound (i.e.
skin flaps) or vascular trauma. Any of these exogenous or endogenous factors predispose the wound to higher
infection rates, and these wounds should only be closed after weighing the risks and benefits.

The length of time the wound has been open and the degree of contamination of the wound are both related to
infection risk. An open wound allows access of normal skin flora to tissues now unprotected by the skin. The
longer a wound is open, the more likely that a significant amount of bacteria have gained access to the wound.
Likewise, the degree of contamination of the wound obviously plays a role in infection. Wounds made by a
sterile razor are less likely to deposit a significant amount of bacteria in the wound than those made by a rusty
nail from barnyard.

Finally the risk of infection with suturing must then be weighed against the functional and/or cosmetic effects of
not closing the wound. For such reasons, large lacerations of the face almost always undergo primary closure,
whereas wounds of the trunk and extremities may not be deemed as urgent to close.

Take Home Message


Balancing the improved healing from wound closure and the risk of infection from closure determines if we
should suture the wound
After 8 hours of being open, the risk of infection generally rises
Some wounds have higher infection risks and some lower. This is related to the degree of wound
contamination and the vascularity of the injured tissues

What to Do If the Infection Risk Is Deemed Too High.


So what to do if the risk of infection is deemed too high to close a wound? Well, there are two options. The first
is healing by secondary intention. In this case the wound is cleaned and bandaged and heals by granulation.
This does increase the size of the scar, but may be appropriate for wounds that have too much tissue loss to
suture or for wounds with high infective risks and low cosmetic importance.

The second option is delayed primary closure. This technique is useful for wounds that cosmetically or
functionally need closure, but the risk of infection is deemed too high to close at the time of presentation. In
delayed primary closure, the wound is cleaned and dead tissue debrided. The wound is then packed with
nonstick sterile gauze and bandaged. The patient changes the dressing daily and the wound is reexamined a
few days after the injury. If there are no signs of infection, the wound is then sutured closed. The results of
delayed primary closure are as good as that of primary closure. The obvious downside of such a technique is
the necessity of daily dressing changes and multiple visits to a health care provider.

Take Home Message


There are multiple ways we can manage a wound
Primary closure means we suture the wound when we see the patient
Delayed primary closure means we suture the wound a few days after the injury, if it isnt showing signs of
infection.
Healing by secondary intention is cleaning the wound and allowing it to heal on its own

I-7
The Goal of the Stitch
Yes, youve gathered that the role of suturing is to close wounds. Wounds with good apposition of tissue heal
faster and better than those that dont. But there are a couple of concepts you need to understand before you
embark on suturing the heck out of everyone in the emergency room.

The first goal of the stitch is tissue apposition and wound eversion. In other words we want to bring all the
tissues together to let the process of healing occur. This concept is important as it helps dictate the depth of
our stitch. Consider the cross section of a laceration shown below:

Figure 1

Note that the wound traverses the dermis and into the subcutaneous tissue. The goal of suture is to
approximate all of this tissue. A properly placed suture will pull the tissues together as demonstrated below:
Tissue apposition and wound eversion can be achieved by placing the needle at ninety degrees to the skin with
the wrist in pronation. The wrist is then supinated following the curve of the needle as the needle and stitch
move through the tissue.

Figure 2
Taken From: Wounds and Lacerations Emergency Care and Closure, page 130, Alexander Trott MD

A common error in suturing is to close the wound too superficially as shown in figure 3:

Figure 3
Fig 1 & 3 - Illustrations by Dr. Ian Rigby used with permission.

I-8
This leaves a space of unopposed tissue under the stitch. Not only does such an area heal poorly, but it is also
a potential site for infection. Selection of a correctly sized suture needle will alleviate this problem. This is
discussed shortly.

The other goal of suturing is to create mild eversion of the edges of the wound. As wounds heal, the scar tends
to flatten. Thus, a laceration that is closed with slight eversion (i.e. the skin puckered up slightly) will eventually
heal flat. Again, proper sizing of you suture needle will assist in this.

Stitches should be placed approximately 1 cm apart on most wounds. Wounds under a lot of tension will
require more closely placed stitches or even deep buried absorbable sutures to bring the wound together.
Remember we want to bring all the tissue together so it can heal nicely.

Finally, a common mistake is pulling your stitches too tight. Although you want the tissues to join together and
evert slightly, it is detrimental to mash the tissues together forcefully. Remember you want apposition, not
strangulation!

Figure 4
Taken From: Wounds and Lacerations Emergency Care and Closure, page 127, Alexander Trott MD

Take Home Message the Goal of the Stitch


The stitch should bring together both the subcutaneous tissue and the dermis
Ideally we would like to cause some mild eversion of the skin so the wound heals flat
A stitch that passes under the base of the wound will accomplish both of the above objectives
Place the needle 90 degrees to the skin surface and pronate and supinate the wrist to follow the needles
curve
Stitches should generally be about 1cm apart from each other. You may need to modify this distance, though,
in order to get the tissues to oppose nicely.
When tying the suture our aim is to firmly oppose the tissue edges but not to strangle them with force.

I-9
Layered Closure
Many wounds or incisions can be closed with a simple interrupted stitch, however it is not uncommon for
wounds to require approximation of up to 3 layers: fascia, subcutaneous tissue and skin. With deeper wounds
the fascia is sutured as muscle is too friable to hold a suture.

Closure of individual layers obliterates dead space which can harbor blood or exudate and lead to infection. In
gaping wounds layered closure prevents scarring and deformation of the wound surface as well as reducing
skin tension. Typically an absorbable suture is used for deeper layer closure.
Multilayer closure is not required in the scalp, extremities, wounds without tension, or poor vascular supply.

Deep Suture Placement


Standard sterile technique, appropriate draping, exposure and illumination as with any wound closure. The
suture enters the bottom of the wound and in a subcutaneous closure exits in the dermis on the same side. The
needle is then pulled through to enter at a matching point on the dermis of the opposite side of the wound and
exit through the bottom of the wound.

Pull the 2 tails of the suture in the same direction of the axis of the wound and tie the knot with square knots
using an instrument tie. This technique produces a buried knot at the bottom of the wound which prevents a
painful palpable mass beneath the skin surface.

Figure 5

Taken From: Wounds and Lacerations Emergency Care and Closure, page 130, Alexander Trott MD

I - 10
Horizontal Mattress Suture
The horizontal mattress suture is a suture employed to help evert skin edges, relieve tension on wounds, useful
for fragile skin and can be used as a highly effective hemostatic suture enclosing a bleeding vessel. The eyelid,
dorsum of the hand, foot or finger are areas where placement of horizontal mattress sutures can be considered
as well. Care must be taken not to tie the sutures too tightly as a larger area of tissue is susceptible to ischemia.

The same basic principles apply as for simple interrupted sutures with placement of needle in driver and 90
degree needle entry to skin.
The horizontal mattress is initiated by placing the needle about 0.5cm from skin edge exiting the wound on the
opposite side. The needle is then placed about 0.5 cm further down the wound edge and passes from the far
side of the wound to the original near side essentially creating a square. The suture is tied on the side where
the needle first passed using square knots to secure the suture.

Figure 6
Taken From: Wounds and Lacerations Emergency Care and Closure, page 129, Alexander Trott MD

Vertical Mattress Suture:


The suture is placed initially like a simple suture with initial bite 1.0 to 1.5 cm from wound edge and a
corresponding bite on opposite side of wound. The needle is then reversed and returns in the opposite direction
for a smaller 1.0 to 2.0 mm bite at the epidermal /dermal edge to approximate the epidermal layer. The vertical
mattress is also useful for skin eversion especially areas of lax skin -dorsum of hand etc.

Taken From: Wounds and Lacerations Emergency


Care and Closure, page 129, Alexander Trott MD

I - 11
Tetanus! Were Always Asking About Tetanus!

One of the routine questions in wound care is whether or not the patients tetanus immunizations are up to date.
Recall that tetanus is a nasty, but preventable disease caused by Clostridium tetani. Fortunately, it is also
extremely rare in North America. We have two weapons in our arsenal against tetanus. The first is
immunization. In Canada, most children receive tetanus toxoid in infancy (at 2, 4, 6, and 18 months and at 5
years old). A final booster is then provided at 14-16 years of age. Tetanus immunization confers approximately
10 years of immunity. If patients have clean wounds but it has been greater than 10 years since their last
immunization, we provide them with a tetanus toxoid booster (Td 0.5cc IM) to update their immunity. When we
do this, bear in mind that this is only protecting the patient from their next exposure to tetanus, and does
nothing for this exposure.

Not all children and adults have received a full set of tetanus immunizations. This is particularly true for new
immigrants and low socioeconomic populations. In persons who may not have current tetanus immunity (i.e.
greater than 10 years since last tetanus immunization, or have not completed a primary series of
immunizations) and have heavily contaminated, tetanus prone wounds, we enlist our other weapon in the war
on tetanus. This is tetanus immune globulin (TIG). This compound is an antibody to C. tetani, and is used to
mop up any potential infection in non-immune patients. It is given as 250 IU IM. Here is a review of tetanus
prevention is:

Clean Minor Wounds All Other Wounds


# of Previous
Doses of Td TIG Td TIG
Tetanus Toxoid
Unknown or less Yes No Yes Yes
than 3 doses
Greater than 3 No* No No** No
doses
* Yes if >10 years since last immunization
** Yes if > 5 years since last immunization

IV. Local Anesthetic


Doing a great job of suturing begins by provided good anesthesia for the patient. The most common technique
for this is local wound infiltration and is demonstrated in the online module. Local anesthetic is administrated
with the smallest needle available preferably a 27 or 30 gauge. Smaller needles are less painful and cause less
damage to the wound, however, it is very difficult to draw up your anesthetic with such a small needle.
Therefore, it is common practice to draw up the anesthetic with an 18 gauge needle and then replacing this
needle with the smaller needle for wound infiltration. To avoid causing more trauma to the skin and to minimize
pain, the anesthetic should be delivered into the wound and not through the intact surrounding skin. Discomfort
can be minimized by using room temperature anesthetic and injecting slowly. Before proceeding with suturing it
is important to ensure adequate anesthetic has been employed. This can be confirmed by testing sensation
with forceps. Once you are satisfied that the wound is adequately anesthetized, you should dispose of your
anesthesia syringe and needles immediately to prevent leaving any uncapped sharps on your tray.

Local anesthetics function by disrupting conduction of nerve signals from anesthetized tissue (blocks sodium
channel conduction for all you budding physiologists out there). The most commonly used local anesthetic is
lidocaine. Lidocaine is found in most hospital wards, operating rooms and emergency departments. Local
infiltration of lidocaine into and around a wound provides anesthesia within seconds, and its effects last
upwards of an hour. There are two formulations of lidocaine; those with epinephrine and those without
epinephrine. The addition of epinephrine causes vasoconstriction of the tissues and provides us with a less

I - 12
bloody field to work in. This is helpful in most wounds, but does need to be used with care in areas of poor
blood supply as it can cause ischemic necrosis. Although somewhat controversial, Epinephrine should NOT be
used in the following areas:
(a) Nose
(b) Ears
(c) Digits
(d) Penis

Those of you who are aspiring physiologists may also remember that lidocaine is actually an anti-arrhythmic
drug (a Ib Sodium Channel Blocker). Thus, patients who receive too much lidocaine can get cardiac and
neurological side effects. Oops, thats not good! The toxic dose of lidocaine depends on whether or not
epinephrine is mixed with it. The vasoconstriction caused by epinephrine slows the systemic absorption of
lidocaine, allowing for larger doses to be used. The maximal dose for lidocaine with epinephrine is 7mg/kg,
while the maximal dose for lidocaine without epinephrine is 5mg/kg. Now this is good examination material, so
lets do an example.

Lets say you are going to provide a local anesthetic for a 70 kg patient with a large wound on his leg. You are
going to use lidocaine with epinephrine, so you know that his maximal dose is 7mg/kg. Doing the math you
realize that this patient can get 490mg of lidocaine (70kg x 7mg/kg). But the bottle of lidocaine you have reads:
1% lidocaine with epinephrine. Help! How much of this can you use?
Well, 1% equates to 1gram in 100ml or 10mg/mL (and 2% is 20mg/mL, etc.). You know your patient can get
490mg of lidocaine. Thus he can get 49mL of lidocaine with epinephrine before becoming at risk for toxicity
(490mg 10mg/mL). As you inject any anesthetic you should withdraw to make sure you are not injecting the
anesthetic into a blood vessel. If you withdraw the plunger and notice blood in the syringe, chances are you are
in a small vessel. Simply repositioning your needle should remedy this problem

Take Home Message


Lidocaine works by blocking sodium channels in the sensory nerves
Lidocaine comes in mixtures with and without epinephrine
You should not use the lidocaine with epinephrine when working on the nose, ears, digits or penis.
Lidocaine can be toxic to the brain and the heart. The maximal doses you can use of lidocaine is 5mg/kg. If
you are using lidocaine with epinephrine the maximal dose is 7 mg/kg.

V. Wound Preparation
Wound preparation involves 6 steps. The first four steps can be done without being sterile.
1) Preparation: prepare by opening your sterile tray. Gather the supplies you will need including sterile gloves,
a syringe (3-5ml) for anesthesia, an 18 gauge needle to draw up the anesthetic and a 27 or 30 gauge needle to
inject. You may also want a 10-20ml syringe and a splash guard for wound irrigation. You will also need a
cleaning solution as described below. Pour this into your sterile container. Select your suture of choice, open
it, and drop the sterile suture cassette on your sterile tray.
2) Optimizing patient comfort and your own comfort: Make sure the patient is comfortable. If you anticipate a
prolonged closure, make sure the patient is lying down and that you are seated comfortably without putting
strain on yourself. Also if you anticipate the patient may become presyncopal, ensure the patient is lying down
or you will have more wounds to fix at the end. Ensure proper lighting.
3) Satisfactory anesthesia: this was discussed above. Dont forget to dispose of your anesthesia needle
immediately. You do not want to have any exposed sharps on your tray

I - 13
4) Adequate cleansing: For cleaning non-contaminated wounds, simply use sterile saline. Some people
choose to use commercially available cleaning solutions such as poviodine or chlorhexidine. These are great
for cleaning intact skin but can be toxic to the devitalized tissue in the wound itself. Recent research has even
suggested simply using tap water to clean the wound is as good as any other alternative. It is common practice
in the emergency department to have the patient go to the sink and clean the wound with tap water and soap,
then clean the wound with normal saline. Deeper wounds should be irrigated with normal saline. This can be
accomplished with your 10-20ml syringe and a splash guard. This is demonstrated in the online module. Good
irrigation markedly reduces infection rates. Wounds that are grossly contaminated will require extra effort to
remove all debris. Dead tissue should be removed with a scalpel or scissors.
5) Sterile gloving and sterile technique: Up until this point in the procedure, it is ok to have been non-sterile.
The appropriate method for putting on sterile gloves is demonstrated in the online module. Once you have put
on your sterile gloves it is important to maintain sterile technique. You should not touch anything that is not
sterile.
6) Draping the wound: Your sterile tray should have adequate sterile drapes. The larger the sterile field, the
better. This makes it less likely for you to contaminate your sterile field and also gives you more room to leave
your sterile instruments (not sharps) in close proximity to the wound.

VI. Choosing Your Suture


Sutures come in many makes, but are generally divided into absorbable and non-absorbable. Absorbable
sutures are broken down by the body and thus do not have to be removed. This can be useful, but absorbable
sutures generally have higher infection rates and worse scarring than non-absorbable sutures. As you can
imagine, non-absorbable sutures do not break down and need removal. Commonly used varieties of each
include:
Absorbable Sutures Non-Absorbable Sutures
Chromic Gut Prolene (polypropylene)
Vicryl (Polyglactin) Ethilon (nylon)
Surgical Silk

When placing sutures inside tissues where they cannot be removed, absorbable sutures are obviously a must.
For most simple lacerations we manage, non-absorbable sutures are best due to lower scarring and infection
rates. For children, absorbable sutures are sometimes placed to limit the anxiety of a return visit to a health care
provider for suture removal.
After choosing the material for your suture, you must then choose the size of the suture. Suture size is done
with the O system. For each extra O, the suture thread gets thinner. As such a 6-0 suture is much thinner
than a 4-0 suture. General guidelines for sizing in standard wound care are:

Body Part Suture Size


Wounds under tension of torso/trunk 3-0 or 4-0
Wounds of the trunk or extremities 4-0 or 5-0
Scalp 2-0 or 3-0
Facial wounds 5-0 or 6-0
Fine wounds (i.e. eyelid) 6-0

And just when you think youve thought too much about your suture, you must next choose a suture needle
size. The sizes of the needles are shown on the suture package. When correctly sizing the needle, think back
to the goal of your suture. Ideally you would like to pass the needle through the skin and completely under the
base of the wound. Figure 2 (way back) demonstrates this. By doing this you provide apposition of all the
tissues of the wound. Furthermore, by encompassing the entire wound, you produce eversion the edges of the

I - 14
skin which is your second goal of the suture. We wont spend any time on the huge variations of needles and
sizes, but by seeing the needle size you should select a needle that will be large enough to encircle the wound.

The correct method for performing an instrument tie is illustrated in the online video

VII. Patient Instructions


Once youve finished your suturing, the patient needs a couple of pieces of information. First off, when to have
the sutures removed. A guideline is:

Place of Wound Days Until Removal


Face 3 5 Days
Extremity/Trunk 7 10 Days
Over a Joint 10 14 Days

Secondly, the patient needs some care instructions. They should be instructed to keep the wound clean and
dry for 48 hours. If they have dressings on the wound, it should be changed daily. Polysporin can be placed on
the wound to prevent infections. Finally, if there are signs of infection (i.e. 3 ps: pain, pus, pink) they need to be
reevaluated by a physician. If the wound is healing properly, the pain should improve daily. However a sudden
increase in the pain may indicate an occult infection. Pus is an obvious indicator of infection. Pink refers to the
color around the wound. If the skin around the wound begins to turn dark pink, this may be an indication of
celluitis, a superficial skin infection.

Patients are often concerned about potential scars especially for face wounds. There are numerous ways to
minimize a scar: placing extra sutures, removing them in 5 days or less, using a smaller suture and preventing
infections. Patients should also be made aware that scars tend to tan more so than the rest of your body and
that the scar wont take on its final appearance for up to two years. Therefore patients should be instructed to
cover the facial wound with sunscreen or wear a hat to minimize the scar from taking on deeper pigmentation
than the rest of the skin for at least 2 years.

Take Home Message


Use commercial skin cleaners around the wound only
Use irrigation with sterile saline or water to clean the inside of the wound
A wound should be closed with proper sterile technique
Sutures come in two general varieties: absorbable and non-absorbable
Sutures are sized with an O system. The more Os you have, the thinner the suture thread is.
The needle of your suture can come in many sizes. In general, select one that will pass through the base of
the wound (remember the objectives of the stitch).
Non absorbable sutures need to be removed. When, depends on where the wound is located.

VIII. Dont Run With Scissors (or other sharps)!


Alright, you now have the low down on the theoretical aspects around suturing. Its time for you to complete the
online demonstration of suture techniques. It is available on the Osler podcast blog.
We would like you to pay particular attention, though, to how the sharps are handled in the module. When

I - 15
doing suturing, we are using many needles, both for the provision of anesthetic and in the actual suturing itself.
Unfortunately, medical students are at very high risk for sharps related injury while learning and performing
procedures such as suturing. Such injuries are a form of body fluid exposure and this puts students at risk for
blood borne infections such as hepatitis and HIV.

There are a couple of ways to minimize your risk. First, a large majority of injuries occur when recapping
needles. So, if at all possible, avoid recapping. If you must recap a needle, utilize a one handed scoop
technique. This technique has been shown to markedly reduce needle related injuries to health care workers.

There is also a risk is from the suture needle. Notice in the demonstration that the needle is always grasped
with an instrument, and not with the fingers. The only time this does not occur is during the instrument tie.
Notice here that the physician grasps the base of the needle prior to starting the tie. This provides him with
control of the needle during the tying process so he is not injured with it.

Finally, when the procedure is done, ensure that all sharps are placed in the appropriate yellow sharps
container. This is your job, and leaving sharps on the suture tray exposes other members of the health care
team to injuries and infections. Take a little extra time and ensure all sharps are properly disposed of at the end
of your procedure!

IX. Needlestick Injuries


Despite careful technique, sometimes sharps related injuries do happen. So what do you do if you get poked?
First, interrupt your procedure and go and wash your wound well with copious amounts of water and soap. If
this occurs in the operating room, you will need to scrub out of the procedure.

Next, you need to report the incident. Inform your attending physician or the charge nurse of the injury. Youll
likely be directed to Occupational Health and Safety (OHS) for evaluation or to the Emergency Department if it
is after hours. There are a couple of reasons to do this. First of all is for your own health! OHS will then
undertake a process of determining your level of risk for infective illnesses from this exposure. They will
coordinate things such as rapid HIV testing of the patient and will counsel you on what preventive methods are
available for you and if they are necessary.

The other reason to report your exposure is financial. Part of OHSs job is to document that at the time of
exposure you are free of blood borne infections. Although it is unlikely you will contract an infection from your
exposure, it is possible. And proof of being healthy at the time of exposure is useful if you need to get
compensation for a work related illness. This cant happen if the exposure is not documented, so be sure to
report any significant injury.

Take Home Message

Avoid sharps related injury


Avoid recapping needles
If you must recap the needle, use the scoop technique
Ensure that you personally dispose of all sharps in the yellow sharps containers
If you do suffer a sharps injury, wash the wound copiously with soap and water
Report any such injury to the charge nurse or attending physician. They will undertake a risk assessment of
the patient on your behalf
Seek medical attention through Occupational Health and Safety or through the Emergency Department (if
after hours).

I - 16
X. Have Fun With It
Despite having read about these injuries, you should know that the risk is low when you handle your sharps
appropriately. All in all, suturing is fun! So go through the online module and see how its done. Well see you at
the skills session, and get you ready to be suturing in the ED, the OR and on the wards!

Please view the online video module available in the Osler podcast blog or on the Core
site which demonstrates the steps discussed here, and then take a few minutes to
complete the attached open-book quiz prior to your small group session. Please bring
your quiz with you and submit it to your preceptor.

I - 17
Module #1 Suturing

Open Book Quiz bring completed quiz to your small group session.
(1) When describing a suture
(a) 5-0 is thicker than 6-0
(b) 6-0 is stronger than 5-0
(c) 3-0 is longer than 4-0
(d) 4-0 is thicker than 3-0

(2) Given the average wound, at what time after injury does the risk of infection markedly rise?
(a) Greater than 4 hours que in
(b) Greater than 8 hours
(c) Greater than 12 hours
(d) Time does not make any difference in infection rates

(3) Proper sterile technique includes which of the following?


(a) Sterile gloves
(b) Sterile water to clean wound
(c) Draping of the wound
(d) Sterile needle to anesthetize wound
(e) All of the above

(4) The goal of the stitch is to:


(a) Invert the wound edges
(b) Cause apposition of tissue edges
(c) Close only the superficial portion of a wound
(d) Enhance emergency physicians income

(5) The maximum ccs you could use of 1% lidocaine with epinephrine in a 90kg adult would be:
(a) 30 cc
(b) 54 cc
(c) 63 cc
(d) 72 cc

(6) If you need to recap a needle you should:


(a) Use the scooping technique
(b) Not recap the needle and take it off while wearing gloves
(c) Pull the needle off while pointed away from yourself
(d) It really does not matter how you do this.

(7) Things that influence wound healing are:


(a) Location of the wound
(b) Patient co-morbidities
(c) Time between injury and wound closure
(d) All of the above

I - 18
Suturing Quiz - continued

(8) Delayed primary closure refers to


(a) Closing the wound days after the injury if there are no signs of infection
(b) Letting the wound heal without any intervention
(c) Closing the wound between 8-12 hours after the injury
(d) When patients present 24-36 hours after their injury.

(9) Which of the following is an absorbable suture?


(a) Ethilon
(b) Silk
(c) Prolene
(d) Vicryl

(10) You poke yourself with a blood-contaminated needle. Which of the following is not an infection that you
should worry about?
(a) Giardia
(b) HIV
(c) Hepatitis B
(d) Hepatitis C

I - 19
Procedural Skills - Module #2
Starting an IV and Intraosseous Access

OBJECTIVES:
Upon completion of this module, students will be able to do the following independently:
Identify the indications for initiation of IV therapy
Describe the common complications of IV placement
Identify the most common sites for placement of an IV
Describe the process of preparing an IV site and for placement of the IV
Correctly demonstrate placement of an IV on a mannequin
Describe the process of performing venipuncture
Correctly perform venipuncture on a mannequin
Describe the appropriate technique for the handling of sharps and identify those practices that place the
health care provider at increased risk of a sharps-related injury

PART I IV PLACEMENT

Indications / Contra-indications / Complications


IV placement, while commonly considered routine for hospitalized patients, should still receive some
consideration prior to initiation. The most common indications for the placement of an IV include the following
Administration of IV fluids (either maintenance or for resuscitation)
Administration of medications or blood products
As a precaution in patients at risk of sudden circulatory collapse requiring resuscitation (eg. GI
bleeding, labor , acute coronary syndromes etc}
Another portal for withdrawing blood for diagnostic studies
A wide variety of fluids can be run through an IV, and the desired solution needs to be specified, as well as the
rate. Options for fluid include 0.9% or Normal Saline (NS), Half-Normal Saline (1/2NS), Ringers Lactate and
Dextrose containing fluids. Obviously the choice will depend in large part upon the clinical setting.

Contraindications to IV placement have more to do with IV site selection than anything else, as there are no true
contraindications to obtaining IV access. In selecting a site, start distally, so that you may move more
proximally if the attempt is unsuccessful. The dorsum of the hand and radial aspect of the wrist are often good
sites for smaller IV placement. More proximal sites, such as the antecubital fossa are favoured for patients
requiring larger fluid infusions. Avoid placing an IV over a joint surface if possible, as potential kinking and
discomfort can occur when joint flexed or extended. Non-dominant arms are preferable to dominant arms when
given the option. Upper extremities are generally favoured due to the lower risk of phlebitis and DVT. A take
what you can get approach is required in patients who present vascular access challenges, such as IV drug
users and chemotherapy patients. In infants, scalp veins often offer the best chance of success. A potential IV
site should be considered contraindicated if any of the following apply:
Evidence of overlying infection of the skin
Trauma to the involved limb
Imminently planned surgery to the involved limb
Proximal venous obstruction
Paresis of the involved limb
Presence of an AV fistula (i.e. for dialysis)

I - 20
Complications of IV placement are generally minor. Phlebitis at the site is reasonably common, and as a result,
IV sites should be changed on hospitalized patients every 3 days. The other common problems involve the
creation of hematomas and bruising, which is often easy to do in patients on blood thinners, or in the elderly
who have more fragile tissues and the risk of introducing infection. A more serious potential complications
relates to the risk of extravasation of the IV fluid or medication from a running IV. Depending on the toxicity of
the medication in question, this could present a threat to the extremity involved.

Take Home Message


Think about whether a patient really needs that IV
Order the type and rate of fluid at the same time as the IV
The best site is usually on the non-dominant arm
When looking for a site, start distally, work proximally
When starting an IV, look at the limb and think Is there anything abnormal about this limb
that should make me put the IV site somewhere else?

Preparation / Equipment
Prior to inserting an IV, ensure that you have everything you will need at the bedside with you (see Figure 1.
This will include the IV fluid and tubing, the catheter itself, something to clean the skin, a tourniquet, tape to
secure the IV in place, and a sterile dressing to
place over it. Gloves should be worn for the entire
procedure but do not need to be sterile.

Once a site has been selected, the site must be


disinfected. This can be done with any of iodine,
alcohol or chlorhexidine. Skin prep should be
done in a circular manner, starting at the inside
and working outwards. Do not shave the area, as
this may cause micro-abrasions, which facilitate
the development of infection. If the site is
excessively hairy, one could trim some of the hair
away (or find another site).

Figure 1: Equipment for IV Insertion


Photo by Dr. Ian Rigby; used with permission

Placement of a tourniquet is key to successful IV insertion. The purpose of the tourniquet is to distend the
veins, thus making them bigger and easier to cannulate. If a site is selected in the forearm or hand, place the
tourniquet approximately 3 inches below the AC fossa. If the IV is to be placed in the AC fossa, apply the
tourniquet approximately half way up the biceps. Remember that the point of the tourniquet is to obstruct
venous outflow causing distention of the veins, without obstructing arterial flow, which you are depending to fill
the vein. If you cant feel a distal pulse, the tourniquet is on too tight (although the patient will likely already
have told you so). Also if the tourniquet is significantly painful for the patient it is probably on too tight, or is
pinching somewhere.

Once the tourniquet is in place, a few other things can be done to make the veins bigger and easier to
cannulate. Ask the patient to clench and release their fist a few times. This will encourage more blood to flow
into the veins. Also, you can try tapping gently on the vein, which will tend to make it engorge somewhat. Be
careful, though, as hitting it too hard will cause vasoconstriction, and then you need to start over. If patients are

I - 21
cold or peripherally shut down, a warm blanket wrapped around the extremity for 15 minutes may help the veins
to dilate.
The next step is to select an appropriate IV catheter. Note that they come in a wide variety of sizes depending
on the patient characteristics. A 16G catheter may be perfect for the 40 year old man bleeding from his
esophageal varices, but entirely impractical for the 80 year old lady who recently finished chemotherapy and
has become dehydrated. In adult patients, the range of sizes you will most commonly see used is between 16
and 25 gauge needles. Bigger and smaller exist, but are used much less often.
You are now ready to put in the IV. Remember that you are now handling sharps and should definitely have
gloves on. Also, make sure not to re-palpate the IV site, or it will no longer be sterile.

Take Home Message


Sterilize the area with iodine, alcohol or chlorhexidine and allow it to air dry
Do not put the tourniquet too tightly, or it wont do what you want it to do
Pick an IV size that is appropriate to the site and the patient

Placing the Needle


The process of actually placing the IV can be thought of in terms of two steps. The first is getting the cannulae
into the lumen of the vein. The second is advancing it proximally. Most beginners find the first step easier than
the second.
Before even attempting to puncture the skin, it is important to immobilize the vein as much as possible. This is
because peripheral veins have a tendency to roll away from the IV catheter as it approaches them. Two
things can be done to address this problem. The first it to pull the skin (and with it, the subcutaneous tissues)
taught prior to introducing the needle. With IV starts in the hand, this is fairly easy to do by flexing the wrist and
pulling downwards on the skin overlying the MCPs with your non-dominant hand (i.e. the one not holding the IV
needle). The other thing that can be done to make getting the IV needle into the vein easier is to take
advantage of bifurcations in the veins themselves. The veins tend to be secured to the underlying tissues at
their branch points, so they will tend to roll less. It is import to note, however, that you shouldnt introduce the IV
directly into the bifurcation itself, as this causes undue trauma to the vein. Always try to introduce the needle
just proximal to the bifurcation. This will also help you avoid those pesky valves.
To introduce the catheter, hold it bevel up, at an angle of between 10 and 25 degrees relative to the skin. The
shallower the vein, the less your angle should be. Advance the catheter until it is within the lumen of the vein.
Ideally, this should be done in a single motion. You will know you are in the lumen when you see blood
flashback in the hub of the needle. Some people also report feeling a pop when the needle penetrates the
vessel wall. The biggest trick at this point is not advancing the
needle through the far wall of the vessel and out into the tissues.
Remember that when you see flashback, the tip of the metal stylet
is within the vein. This does not mean, however, that the catheter is
within the vein, because the stylet sticks out ~2 mm past the end of
the catheter. (see Figure 2) The stylet / catheter unit therefore
needs to be advanced a little bit further before the stylet is
withdrawn and the catheter advanced. While keeping the vein
immobilized, lower the catheter (flatten it out) until it is nearly
parallel to the skin and then advance it slightly. Figure 2: IV Catheter Tip
The pushing off method is the most commonly employed technique Photo by Dr. Ian Rigby used with permission
for advancing the catheter. After introducing the catheter and
achieving flashback of blood, advance the catheter a little further to

I - 22
ensure that you are securely within the lumen of the vein. Once the stylet / catheter is clearly within the lumen,
use the index finger to advance the catheter while holding the stylet steady or retracting it slightly with the
thumb and middle finger (see figure 3). The thumb and middle finger do not move, only the index finger moves
forward as it advanced the catheter. Once the catheter is advanced all the way to its hub, remove the stylet,
then the tourniquet and attach the IV tubing.

Figure 3: Pushing Off Method of Advancing IV

Take Home Message


Immobilize the vein in order to maximize chances of success
Enter skin at 10-25 degree angle relative to the skin.
When blood is seen in the hub of the needle, the tip of the stylet is in the lumen. Make sure to
advance the catheter a little further to ensure catheter is in the lumen as well.
Lower the catheter parallel to the skin before advancing it
The catheter can be advanced by floating it on a running IV or pushing it off the stylet

Securing the IV & Disposing of Sharps


Securing the IV is fairly straightforward. The goal of the process is two-fold: securing the site so that the
catheter does not dislodge and keeping the site clean. With the IV catheter in place and the tubing connected,
place a strip of tape sticky side up under the hub of the catheter. Bend the tape back covering the wings and
run alongside the direction of the IV so the site remains visible (see Figure 4 below). Then put a piece of tape
perpendicular over the hub. Next, apply a sterile dressing to the IV site, making sure to place it in such a way
as to allow the IV tubing to be changed without having to remove the dressing. Lastly, bend the IV tubing back
in a loop, so that you can secure the IV tubing more proximally on the limb in question with a couple of strips of
tape. Another alternative to securing the IV is a sterile, clear adhesive dressing as demonstrated in the online
video module.

I - 23
Figure 4: Securing the IV IV Connector Tubing

Sharps disposal and handling should remain foremost in your mind throughout the process. The most
important thing to do is to remain conscious of the location of your sharps at all times. After removing the stylet
from the catheter, you will want to use one finger to occlude the vein more proximally to minimize the amount of
bleeding while you attach the IV tubing. Do not let the fact that you must concentrate on that activity allow you
to lose track of the stylet. If you are using a tray, place the stylet in clear sight on the tray. If you have a
bedside sharps disposal box, place it directly into the box before proceeding further. Even if the IV attempt
fails, you will not need to re-use the stylet. Do not, under any circumstances, place the stylet on the patients
bed or sheets. This is an invitation to receive a needlestick injury within a few minutes. Newer IVs have a built
in safety feature to minimize needlestick injuries. As the stylet is withdrawn it is contained within a plastic
chamber as seen in the diagram below

Figure 5: Safer IV alternatives

Take Home Message


Use either tape or a sterile dressing to secure the IV site
Place the dressing so as to allow the IV tubing to be changed easily
Never lose sight of your stylet after removing it from the catheter. If possible dispose of it
directly into a sharps box after removing it from the vein

I - 24
PART II INTRAOSSEOUS ACCESS

Intraosseous (IO) is an alternative route for rapid access to the vascular system and allows for infusion of fluid,
blood and medications. An IO needle can be rapidly inserted in a patient in shock or a patient where peripheral
or central venous access is difficult or impossible. The IO needle is placed into the bone marrow which
functions as a noncollapsible venous access route.

Complications from an IO are rare. Extravasation of fluid is the most common complication. This rarely causes
any problems unless the medication is caustic (calcium chloride, bicarb, etc). . Hematomas at the insertion site
are also possible but uncommon. Infection and osteomyelitis are theoretical risks but are very uncommon as
IOs are usually removed within a few hours (24- 48 hours max). Other rare complications include fat
microemboli, fractures at the insertion site (only in kids if inserted at the growth plate) and compartment
syndrome (again more common in kids if the needle is inserted all the way through the back side of the cortex
and fluid is inserted into the calf muscles).

Intraosseous needles should not be placed where there is overlying cellulitis, in a bone where a previous
attempt has been made or in a rare bone condition osteogenesis imperfecta.

The most common site for insertion of an IO needle is in the proximal tibia. Other sites include distal tibia, distal
femur, superior iliac crest and proximal humerus.

Proper technique for proximal tibia insertion is as follows: Locate the tibial tuberosity and move two centimeters
distal and medial to the tuberosity for insertion site. Once the appropriate site is landmarked use sterile
technique and stabilize the limb with one hand. The drill is held at a 90 degree angle or tilted slightly caudally to
avoid growth plate. The needle on the catheter is placed into the skin down to the bone (without drilling yet).
Once bone is contacted the drill can be engaged.

I - 25
You know you are in the bone marrow when a pop if felt or there is a loss of resistance. Once the catheter is
secured into the bone, the inner cannula is unscrewed and placed in a sharps container. The next step is to
confirm placement in the marrow. A 10ml saline syringe is attached to the cannula and either saline should
infuse easily or blood should be aspirated. The next step after confirming the catheter is in the right place is to
secure the IO to the patient using one of the provided securing devices. The IO can then be connected to an IV
using a connector. Infusion of any fluid is very painful and infusion of lidocaine should be considered to reduce
discomfort

Remember that like any skill, your comfort and success with starting an IV or IO will be dependent on how often
and how regularly you practice them. Take advantage of every opportunity you can. Nurses on the wards and
in the ER are often more than happy to provide you the opportunity to practice if you express interest.

Please view the online video module available on the Osler podcast blog or Core site,
which demonstrates the steps discussed here, and then take a few minutes to complete
the attached open-book quiz prior to your small group session. Please bring your quiz
with you and submit it to your preceptor.

I - 26
Module #2 Starting an IV and Intraosseous Access
Open Book Quiz bring completed quiz to your small group session.

1: Identify three indications for the initiation of an IV.

i. ___________________________________________________

ii. ___________________________________________________

iii. ___________________________________________________

2: Describe the ideal site for the establishment of a peripheral IV.

_________________________________________________________

_________________________________________________________

3: Name 3 relative contraindications to using a particular site for placement of an IV.

i. ___________________________________________________

ii. ___________________________________________________

iii. ___________________________________________________

iv. ___________________________________________________

4: What can be done to make veins more prominent for placing the IV

_________________________________________________________

_________________________________________________________

5. Identify two techniques that can be used to prevent veins from rolling.

i. ___________________________________________________

ii. ___________________________________________________

6. The shallower the vein being cannulated, the greater the angle should be between the needle and the skin
when the needle is introduced.

True False

I - 27
Starting an IV and Intraosseous Infusion Quiz Continued

7. Describe one technique for advancing the IV catheter once the tip is inside the vein.

__________________________________________________________

__________________________________________________________

__________________________________________________________

__________________________________________________________

8. Where should sharps be placed during the procedure if no sharps container is readily at hand?
__________________________________________________________

__________________________________________________________

__________________________________________________________

__________________________________________________________

I - 28
Procedural Skills - Module #3
Lumbar Puncture
Introduction
Lumbar puncture (LP) and analysis of cerebrospinal fluid (CSF) has been part of the medical profession for over
one hundred years. This relatively simple procedure is widely employed and provides critical diagnostic data,
yet a surprising number of physicians are poorly educated in regards to when and how this procedure is
performed.

Objectives
By the completion of this teaching unit, the student physician will be able to:

i) List the indications for lumbar puncture


ii) List the contraindications for lumbar puncture
iii) Describe when a CT scan is and is not required prior to lumbar puncture
iv) List and discuss the complications of lumbar puncture
v) Describe the equipment employed for lumbar puncture
vi) Demonstrate a safe method for performing a lumbar puncture on a model
vii) Discuss the appropriate CSF studies to be ordered in cases of suspected meningitis or
subarachnoid hemorrhage

Indications
The indications for lumbar puncture include (common uses in bold):

Diagnosis of central nervous system (CNS) infection


Diagnosis of subarachnoid hemorrhage (SAH)
Infusion of anesthetic, chemotherapy, or contrast agents into the spinal canal
Treatment of idiopathic intracranial hypertension
Evaluation and diagnosis on demyelinating or inflammatory CNS processes

Perhaps the most common indication for an emergency LP is to evaluate the patient for CNS infection. CSF
samples are needed for the diagnosis of bacterial and viral meningitis as well as for numerous other infective
processes such as CNS syphilis, TB, fungal infections and other encephalopathies.

The diagnosis of subarachnoid hemorrhage (SAH) is the other emergency indication for LP. In a SAH, an
aneursym (usually around the circle of Willis) ruptures and blood seeps into the CSF. Because SAHs are an
acute life threatening condition and CTs arent 100% diagnostic, an LP is mandatory after a negative CT when
a SAH is suspected. The LP is needed to check for xanthochromia, which is the breakdown product of blood in
the CSF. Usually a negative LP for xanthochromia and a negative CT is enough to rule out a SAH. If the CT is
negative but the LP is positive for xanthochromia, this is an indication for further imaging (CTA or MRI/MRA) to
look for the aneursym.

I - 29
Contraindications
As with any procedure, there are times when it is not safe to proceed with lumbar puncture. These
contraindications are:
Skin infection at the site of the lumbar puncture
Suspicion of increased intracranial pressure due to a cerebral mass
Uncorrected coagulopathy
Acute spinal cord trauma

The most worrisome contraindication to lumbar puncture is the suspicion of increased intracranial pressure
(ICP) due to a cerebral mass lesion. In the presence of a potential mass lesion (brain tumor, cerebral
hemorrhage, cavernous sinus thrombosis, brain abscesses, epidural or subdural hematomas), patients have an
increased risk of neurologically deteriorating after an LP. As a space-occupying lesion grows, ICP rises. As the
CSF pressure drops in the spinal column, a brain mass may then shift towards the low-pressure outlet (the LP
site). This may lead to either trans-tentorial or uncal herniation and acute neurological deterioration.

As with most invasive procedures, uncorrected coagulopathy is a contraindication to LP. This includes those on
heparin, coumadin, or with clotting defects such as disseminated intravascular coagulation, hemophilia or
thrombocytopenia. Reversal of coumadin can be achieved with Vitamin K and fresh frozen plasma or octaplex,.
Replacement of a hemophiliacs clotting factors or transfusion of platelets to the thrombocytopenic patient would
all allow for safe lumbar puncture in these patients.

To CT or Not To CT? That is the Question:


Increased ICP due to a mass lesion is a contraindication to lumbar puncture. In these cases a CT scan is
recommended prior to LP. Unfortunately, it has been wrongly interpreted by many that all patients require CT
scan prior to their LP. This is not so.

The clinical findings of decreased level of consciousness, focal neurological deficits, or papilledema make a CT
scan necessary prior to an LP. If your patient has a normal level of consciousness, no focal neurological
defects, and you cannot visualize the fundi, it is still safe to proceed with the LP.

Complications
All Procedures have potential complications. The potential complications of this procedure are:
Post LP headache
Post LP back pain
Seeding of infection to the CSF
Epidermoid tumor implantation
Uncal or transtentorial herniation and neurological deterioration
Spinal hematoma
Post lumbar puncture headache is the most common complication of LP. The post LP headache develops in 5
40% of patients undergoing lumbar puncture. It is most common in young healthy females. It is a headache
that begins within 72 hours of LP and usually lasts less than 5 days. The headache is a bilateral pressure or
throbbing that is intensified in the upright position and with coughing/valsalva maneurvers. The headache
usually resolves when the patient is supine.

From a pathophysiological viewpoint, the post LP headache occurs because of the puncture in the dura mater
caused by the LP needle fails to seal. This opening allows for the continued leakage of CSF out of the dura
and this lower pressure allows the brain to shift downward. Traction on the pain sensitive cerebral bridging
vessels, dura and cranial nerves causes the headache. When supine, the pressure column of the CSF is equal
and thus there is no pull on pressure sensitive structures of the brain and the headache resolves.

I - 30
Despite the fact that post LP headache is relatively common especially in young healthy females, there are a
number of ways to minimize post LP headaches in your patient. The first is to use the smallest size spinal
needle possible. The incidence of post LP headache is about 70% with 16 19 gauge needles, 20 40% with
20 22 gauge needles and 5 12% with 24 27 gauge needles. The LP kit carried in our hospitals contains a
22 gauge needle but smaller needles are available in the ED and OR.

Another factor that can be utilized to minimize the chance of post LP headache is the use of a stylet in the
needle. The LP needle itself is hollow and therefore has an internal removable stylet. This internal stylet
prevents skin and other tissue from plugging the needle. Replacement of the stylet into the needle prior to the
needles removal at the end of the procedure has been shown to reduce the incidence of post LP headache by
50%. It is theorized that during CSF collection, a strand of arachnoid fiber may enter the needle and when the
needle is withdrawn the arachnoid strand is pulled out through the dural defect and prevents the puncture in the
dura from completely closing.

Furthermore, the type of needle and the orientation of the needles cutting edge also influence the incidence of
post lumbar puncture headache. There are three main types of needles, pictured below. The Quincke needle
was the first invented and has a beveled cutting tip. The Whitacre and Sprotte are atraumatic or pencil point
needles and have blunt tips with lateral ports for CSF collection.

Figure 1. Spinal needle types


Illustration by: Leeanne Rigby
used with permission

I - 31
When using the Quincke needle, the orientation of the bevel influences the incidence of post lumbar puncture
headache. Post LP headache is reduced by 50% or greater when the bevel is parallel to the dural fibres long
axis. To you and me this means that if the patient is lying in the lateral decubitus position, the flat portion of the
bevel should point up towards the ceiling.

Although the usual LP kit contains a Quincke needle, there is very good evidence that the atraumatic needles
significantly reduce the incidence of post lumbar puncture headache. It is theorized that these needles spread
the dural fibres and cut fewer fibres. This reduces the size of the hole in the dura and reduces the tendency to
develop CSF leak. Numerous studies demonstrate post lumbar puncture headache rates of only 2-6% using
atraumatic needles compared to rates of 18-40% using the same sized Quincke needles.

Therefore, in order to reduce post lumbar puncture headache, we recommend using small, atraumatic LP
needles and replacing the stylet at the end of the procedure.

Also common after lumbar puncture is local back pain at the site of puncture. Approximately one third of
patients will experience some local back discomfort after the procedure, which lasts for a couple of days. This
is due to local soft tissue trauma. In rare cases, if the needle is inserted beyond the subarachnoid space, the
annulus fibrosis may be damaged and the intervertebral disk can herniate. This is very rare.

Thankfully, the remainder of complications from lumbar puncture are much less common. The risk of
introducing organisms into the CSF from a properly performed lumbar puncture is exceedingly small (0.2%).
This can occur with breaks in sterile technique, use of contaminated equipment and placement of the needle
through infected skin.

Epidermoid tumor implantation is a theoretical concern that is very rare but thought to occur when a plug of skin
is carried into the spinal canal where it presents months to years later as an expanding epidermoid tumor. The
use of a stylet with lumbar puncture has made this complication mostly one of historical significance.

Spinal subdural and epidural hematomas are also a rare complication reported in lumbar puncture. It is most
common in those who undergo lumbar puncture while having coagulation abnormalities In spinal subdural
hematomas, patients present with severe low back, radicular pain, sensory loss or weakness hours to days post
LP. These symptoms in a post LP patient warrant aggressive investigation with CT/MRI and an immediate
decompressive laminectomy if a hematoma is present.

The Anatomy of the Lumbar Puncture


Let us review a little anatomy so that you can understand where to do your lumbar puncture. At birth, the
inferior end of the spinal cord is opposite the body of the third lumbar vertebrae (L3). Distal to this point is the
cauda equina and its nerve roots. As the child grows, the vertebral column grows much faster than the spinal
cord itself, and by adulthood, the spinal cord only reaches the inferior border of the L1 vertebra, or the superior
aspect of L2. Distal to this point is the cauda equina. In order to avoid transfixing the spinal cord during LP, the
needle is placed distal to L2. This means our needle enters the subarachnoid space at the level of the mobile
cauda equina.

I - 32
Landmarking the interspace is quite easy, as a line that joins the posterior superior iliac crest will cross the L3-
L4 interspace (as shown below). Thus, one can easily identify the L2-L3, L3-L4, or L4-L5 interspaces, all of
which are suitable for LP.

Figure 2. Land-marking the L3-L4 interspace

The CSF itself resides in the subarachnoid space between the pia mater and the arachnoid mater. In order to
place the needle into the subarachnoid space, the needle passes between two spinous processes and
continues through the interspinal tissues and into the subarachnoid space. The tissues pierced are (in order):
skin, subcutaneous tissue, supraspinal ligament, interspinal ligament, ligamentum flavum, dura mater, the
arachnoid mater and into the subarachnoid space. In patients with a normal BMI, the average distance from
skin to the ligamentum flavum is around 44mm. In obese patients, the average distance is 64mm.

Figure 3. Local spinal anatomy


Illustration by: Leeanne Rigby used with permission

I - 33
Tools of the Trade
We are almost ready to talk on how to do the lumbar puncture. First, an introduction to the equipment you will
be using. The lumbar puncture kit contains: two sterile drapes, three cleaning sponges, a 22 gauge spinal
needle, a small 25 gauge and a larger 22 gauge needle for anesthetic infiltration, a 3cc syringe,, a pressure
manometer with tubing, four collection vials and a Band-Aid. The other tools you may need include sterile
gloves, a facemask, a gown, and chlorhexidine cleaning solution. You will also need 1 or 2 % lidocaine
without epinephrine. The kit should look similar to the photograph below:

Figure 4. LP tray
Photo by: Dr. Ian Rigby used with permission

The Lumbar Puncture


The LP consists of five steps all of which are demonstrated in the online module:
(A) Patient Positioning
(B) Equipment Setup
(C) Anesthesia
(D) The LP
(E) Opening Pressure Measurement and CSF Collection
(A) The first part of a successful lumbar puncture is the positioning of your patient. Your patient should lay in
the left lateral decubitus position with their back facing you at the edge of the bed. In this position, the patient is
encouraged to curl up his shoulders and legs while arching his back like a cat. This position will maximize the
distance between the spinous processes while pulling the spinal cord superiorly (away from the LP site). It is
important at this point to ensure that your patients hips and shoulders are perpendicular to the bed, as rolling of
the shoulders or hips will distort your landmarks and decreases the likelihood of a successful lumbar puncture..
Another acceptable option is to perform the LP while the patient is sitting up while resting their upper body on a
bedside table. The same landmarks are used.

Once your patient is in the proper position, feel the posterior superior iliac crests and imagine a line that
connects them. This line will cross the L3-L4 interspace. You may choose this space or one above or below to

I - 34
use in the adult patient. You can mark this spot by leaving an indentation in the skin. Once the patient is
properly positioned, ensure that you have a stool for sitting and that the bed is raised to a comfortable height for
you to do the procedure.

(B) At this point you put on your mask and gown, open your LP tray, and then don your sterile gloves. If you are
performing the LP on a neonate or child, there is a specific tray for pediatric patients with a smaller LP needle.
It is always helpful to set up the tray before proceeding. Stand the collection tubes in their holders and remove
their caps. Draw your local anesthetic into the provided syringe with the 25 gauge needle, and assemble your
manometer. Have an assistant place chlorhexidine into the provided compartment in the tray.
Once your tray is set, place the white sterile sheet along the bed, slightly under your patient. By keeping the
sheet folded over your hands you will not ruin your sterile gloves. Now apply the chlorhexidine solution to your
patients back using the provided sponge brushes. Work in circular strokes starting at the skin dent you placed
on the patients back and working concentrically outward. Repeat this two more times with the provided sponge
sticks. At this point, pick up the blue sterile drape. Notice that it has a piece of tape on one side that will
adhere to the patient and help hold the drape in place. Place the drape on the patient so the hole exposes the
area you intend to use for the lumbar puncture. Ensure the top of the blue drape rests over your patients hips
as this drape will allow you to palpate the posterior superior iliac crest while remaining sterile if you need to re
landmark your LP site.

(C) You are now ready to provide the anesthesia for the lumbar puncture. Locate your chosen interspace (L3-
L4 if using the posterior superior iliac crest line). Take your anesthetic (with a 25 gauge needle) and locate a
point approximately two-thirds of the way down the interspace (i.e. two thirds the distance caudal to the L3
spinous process if using the L3-L4 interspace). Raise a small skin bleb with your lidocaine here. Proceed to
anesthetize the deeper subcutaneous structures by directing the needle towards the umbilicus. After half of the
lidocaine is administered, switch your needle to the longer 22 gauge needle. Place this needle in the area
previously injected with lidocaine. With this needle anesthetize the deeper tissues in a fan shaped distribution.
This fan shape distribution is needed to anesthetize the recurrent spinal nerves that innervate this area and this
will make the procedure much less painful.

(D) After the anesthetic has been placed, it is time to proceed with the actual LP. Take out your spinal needle,
and check that the stylet slides easily. When doing the LP with the provided spinal needle, ensure that the
notch of the stylet (that bump on the plastic part of the stylet) is facing up to the ceiling. Position the needle at
the site of your anesthetic injection (two thirds distally between the two spinous processes). Your needle should
be parallel to the bed and directed towards your patients umbilicus. If the patient is sitting up the bevel should
parallel to the spinal cord.

Grip the proximal potion of the needle in your left hand for control and with your right hand guide the needle
while holding the stylet in place. Puncturing the skin may require some force. Following the skin puncture,
make sure the needle remains parallel to the bed and aimed slightly cephalad towards the patients umbilicus.
Continue to advance the needle through the subcutaneous tissue and into the supraspinal and interspinal
ligaments. You can tell you are in the interspinal ligament when the LP needle becomes a bit more difficult to
advance because the ligaments are tougher tissue than the subcutaneous fat but the needle should still move
easily without excessive force. Occasionally, while advancing the needle, you will encounter bony resistance.
If this should happen, remove your needle all the way back to the skin and direct the needle slightly more
cephalad. Do not attempt to redirect the needle without withdrawing it to the skin as this may cause excessive
bending of the needle and make brake off the distal part of the needle in the patients back. Also unless you
withdraw the needle completely, the needle will have a tendency to follow the incorrect tract just created by your
previous attempt. As you advance the spinal needle deeper through the ligaments the stylet is periodically
removed to check for flow and then replaced if there is no CSF flow. As the needle gets deeper and closer to
the spinal canal it is ok to leave the stylet out. At this point it is unlikely the needle will get plugged with tissue
and makes the flow of CSF immediately visible. Also, it makes it less likely to insert the LP needle too far

I - 35
forward through the spinal canal into the vertebral plexus or into the posterior part of a vertebral body/disc,
especially in young children.. As you advance the needle farther you should feel a pop. This accompanies the
piercing of the dura mater and if you are in the correct place, CSF should flow freely from the needle.

(E) If you are measuring opening pressure (recommended), your manometer is attached to the spinal needle
and the stopcock opened to allow for CSF to fill the manometer. It is often helpful to have an assistant steady
the top portion of the manometer, so you are free to manipulate the stopcock. Measurement of the opening
pressure ends when the respiratory variation (rise and fall of the fluid meniscus with breathing) ceases. Normal
opening pressures are 5-20 cm H2O. Following the opening pressure measurement, the stopcock is turned so
that the CSF drains out through the spigot and is collected in the four test tubes provided in the LP kit. You
cannot measure opening pressures in patients who have been sitting up for the LP. The pressure will be falsely
elevated. An increased opening pressures suggest increased intracranial pressures from a mass lesion
(neoplasm, hemorrhage or cerebral edema), overproduction of CSF (choroid plexus papilloma) or a defective
outflow mechanism through the ventricles

If you are not measuring opening pressure, then when CSF flow begins, collect the CSF in the sequentially
labeled test tubes (i.e. tube #1 gets CSF first, tube #4 gets CSF last). You need to collect approximately a 1ml
in the first two tubes and .5ml milliliter in the last two tubes.

When the necessary CSF has been collected, the stylet is replaced into the spinal needle and the needle is
withdrawn in one motion. Clean your patients back and put the provided Band-Aid onto their back.
Congratulations, you have just completed the lumbar puncture!

What To Do With This CSF


The recommended studies include cell count with differential, gram stain, culture and sensitivity, glucose,
protein, xanthochromia and viral PCR.

In order to interpret your results, it is important to know about the normal CSF content. The following table
outlines the values seen in a normal LP

Study Normal Value Comment


Opening Pressure 5-20 cm H2O
Appearance Crystal clear Fluid may appear clear with as many as 400
cells/mm3
Xanthochromia None
RBCs 5 per mm3 May be increased in traumatic tap
WBCs 5 per mm3 Exclusively lymphocytes and monocytes
Glucose 60-70% of serum value
or 2.2-3.9 mmol/L
Protein 0.2 0.45 g/L Increased in disease states
Gram Stain and C&S Negative

Normal CSF is crystal clear in appearance, yet up to 400 cells/mm3 can reside in the CSF and the physician will
not see changes in the clarity of the CSF. There are two major reasons for cloudy CSF. The first is the
presence of large numbers of WBCs, which can make the CSF appear turbid or even like pus. The second
reason for CSF discoloration is due to red cells and their breakdown products. Large numbers of RBCs in the
CSF can make the CSF appear very bloody. After the blood has been in the CSF for greater than 12 hours, the
red cells begin to lyse in large quantities producing xanthochromia.. Xanthochromia peaks about 24 hours after
blood enters the CSF and resolves in 3 30 days. The presence of xanthochromia is always pathological.

I - 36
Normal CSF is allowed to have up to 5 RBCs per mm3, albeit it is common to find no RBCs in the CSF. Levels
higher than this suggests a SAH, intracranial bleed or traumatic tap. A traumatic tap occurs when the LP
needle enters a blood vessel while performing the procedure or when the needle has advanced slightly too far
and transfixed the internal vertebral plexus (the more densely packed area of vasculature on the ventral side of
the spinal cord). Differentiating between traumatic tap and SAH is usually fairly easy. As the CSF washes the
needle, the blood will also be washed out and the number of RBCs should decrease from tube 1 to 4.

The normal CSF contains up to 5 WBCs per mm3. These may be either lymphocytes or monocytes. If the CSF
contains more than 5 WBCs or other cell lines, infection is likely. The most worrisome of these is acute
bacterial meningitis. Bacterial meningitis displays a marked pleocytosis ranging between 500 and 20000
WBC/mm3. The differential of these cells demonstrates mostly neutrophils. Meningitis may also be caused by
a variety of viruses. The CSF in these cases demonstrates 10 to 500 WBCs per mm3 with a differential of
mostly lymphocytes and monocytes. To assess for viral infections, the CSF is also sent for Viral PCR. This
takes about 24-48 hours to come back and tests for herpes simplex virus and enteroviruses. Special tests like
West Nile Virus and criptococcal meningitis ( a fungal infection) are available upon special request.

CSF glucose is normally 60-70% of the serum values. Low levels of glucose are commonly seen in CNS
infection and are due to inhibition of the glucose active transport as well as increased utilization of glucose by
the brain and spinal cord. Elevated glucose levels are usually inconsequential and suggest serum
hyperglycemia.

CSF protein usually runs in the 0.2 0.45 g/L range. Increased protein levels are seen in numerous disease
states including meningitis, SAH and some demyelinating neurological conditions.
Gram stain is an invaluable tool in suspected bacterial meningitis. Gram negative diplococci are suggestive of
N. meningitidis. Small gram negative bacilli may indicate H. influenza. Gram positive cocci are suggestive of S.
pneumonia, Streptococcus or Staphylococcus species. Unfortunately, up to 20% of gram stains may be falsely
negative as there are not enough organisms to see.
A review of the CSF values in common disorders shows:

Study Bacterial Meningitis Viral Meningitis SAH


Opening Pressure Often elevated Often elevated Often elevated
Appearance Clear to turbid Often clear Clear to bloody
Xanthochromia Negative Negative Often present
RBCs <5 per mm3 <5 per mm3 >50 per mm3
WBCs Elevated. Many PMNs Elevated. Many Slightly increased
lymphocytes
Glucose Low Normal Normal
Protein Elevated Elevated Elevated
Gram Stain May show organisms Normal Normal

Now that you understand the basis of lumbar puncture and its uses, lets get you to the skills lab!

Please review the online video module available on the Osler podcast blog or Core site,
which demonstrates the steps discussed here, and then take a few minutes to complete
the attached open-book quiz prior to your small group session. Please bring your quiz
with you and submit it to your preceptor.

I - 37
Module #3 - Lumbar Puncture

Open Book Quiz bring completed quiz to your small group session.

This test is multiple choice and is to be completed following the pre-lab reading. There is
only one correct answer per question.
1. The imaginary line connecting the posterior superior iliac crests crosses what spinal interspace?
(a) L1-L2
(b) L3-L4
(c) L5-S1
(d) L4-L5

2. The number and type of white blood cells in normal CSF is


(a) <5 WBC/mm3 all polymorphs
(b) <10 WBC/mm3 all lymphocytes
(c) <5 WBC/mm3 all lymphocytes
(d) <2 WBC/mm3 of any type

3. Which of the following patients does not need a CT-scan prior to lumbar puncture?
(a) A well patient with new onset right arm weakness
(b) A known cancer patient with headache and new onset right arm weakness
(c) An acute trauma patient with headache and bilateral leg weakness
(d) A twelve year old boy with normal exam but fever and nuchal rigidity

4. Which of the following is an absolute contraindication to lumbar puncture?


(a) Uncorrected coagulopathy
(b) Nuchal rigidity
(c) Inability to visualize a patients fundi
(d) A concussion two months ago

5. Lumbar puncture is not used to diagnose which condition?


(a) Subarachnoid hemorrhage
(b) Cerebral abscess
(c) Meningitis
(d) Idiopathic intracranial hypertension

6. What needle type is best used for reducing the incidence of post lumbar puncture headache?
(a) Small blunt tipped needles
(b) Small cutting tipped needles
(c) Medium sized blunt tipped needles
(d) Medium sized cutting tipped needles

I - 38
Lumbar Puncture Quiz continued

7. When advancing the lumbar puncture needle, it should be parallel to the bed and aimed towards the
patients _______________
(a) Sternum
(b) Symphysis pubis
(c) Umbilicus
(d) Neck

8. Which of the following is not a complication of lumbar puncture?


(a) Local back pain
(b) Epidermoid tumour implantation
(c) Olfactory disturbances
(d) Post lumbar puncture headache

9. Which of the following is not a way to reduce post lumbar puncture headaches?
(a) Using a blunt tipped spinal needle
(b) Replacing the stylet in the lumbar puncture needle prior to the needles removal
(c) Keeping the patient on their back for 4 hours after the lumbar puncture
(d) Using a smaller spinal needle

10. Correctly orienting the bevel of a cutting type spinal needle can reduce the incidence of post lumbar
puncture headache. In a patient in the lateral decubitus position, the notch of the stylet should point
_____________-
(a) Up at the ceiling
(b) At the patients right shoulder
(c) Towards the patients head
(d) Towards the patients feet

11. In an adult, the spinal cord may reach as low as what vertebral body?
(a) L4
(b) L2
(c) L5
(d) L3

12. Which of the following gives the correct order of tissues penetrated in a lumbar puncture?
(a) Skin - subcutaneous tissue ligamentum flavum interspinal ligament dura
(b) Skin supraspinal ligament dura ligamentum flavum arachnoid mater
(c) Skin interspinal ligament supraspinal ligament ligamentum flavum dura
(d) Skin supraspinal ligament interpsinal ligament- ligamentum flavum dura arachnoid mater

13. The pop that is often felt in performing a lumbar puncture is caused by the penetration of what tissue?
(a) Dura mater
(b) Subarachnoid space
(c) Intraspinal ligament
(d) Supraspinal ligament

I - 39
Lumbar Puncture Quiz continued

14. If bony resistance is encountered while performing a lumbar puncture, bring the spinal needle back to the
skin and direct it more towards the ______________
(a) Right
(b) Feet
(c) Parallel plane of the back
(d) Head

15. Xanthochromia is caused by


(a) Lysis of red blood cells
(b) Lysis of white blood cells
(c) High concentrations of organisms in the CSF
(d) Acutely traumatic lumbar punctures

1. The color of normal CSF is


(a) Creamy yellow
(b) Slightly pink
(c) Clear straw colored
(d) Crystal clear

I - 40
Procedural Skills Module #4

Basic Airway Management


Introduction
Inadequate ventilation is a life-threatening situation that deprives a person of oxygen and can ultimately lead to
death. It can be due to airway obstruction, inadequate respiratory effort or a combination of both of these. All
medical practitioners need to be able to address these issues as they can occur unexpectedly. Unfortunately,
basic airway management and bag valve mask ventilation is more challenging than it appears, and it takes
practice to be able to accomplish this skill. This teaching module aims to introduce the learner to the
techniques of establishing airway patency and providing non-invasive ventilation. All of the procedures
discussed below are demonstrated in the online module available on CIS.

Objectives
By the end of this module the student will be able to
(1) Identify inadequate ventilation
(2) Open an obstructed airway with a jaw thrust and/or chin lift
(3) Provide positive pressure ventilation with one and two person techniques
(4) Place an oropharyngeal airway in the appropriate patient place this as third and positive pressure
ventilation as 4

Establishing Airway Patency


Begin by assessing your patient for responsiveness. Unconscious patients are at high risk for airway
obstruction. Sounds of gurgling or raspy respirations also indicate partial airway obstruction. In such a case,
consider suctioning the airway to remove foreign material (i.e. vomitus). If the patient is not breathing or
continues to have a partial obstruction, continue with the steps below.

Patients with decreased levels of consciousness often obstruct their airway with as their tongue falls backwards
into the posterior pharynx. Two simple procedures can be used to address such an obstruction.

The first is the chin lift. Using your left hand, place two fingers under the patients mandible and lift upwards.
Your right hand supports the patients head and allows for extension of the head. This movement moves the
patients tongue away from the back of the throat and opens the airway.

The chin lift technique cannot be employed in patients with suspected cervical spine injury. A second technique
can be used to open an obstructed airway and is called the jaw thrust. To accomplish this place your pointer
and middle fingers behind the angles of the patients mandible and push the mandible forward. This, too,
serves to move the tongue from the posterior pharynx and can be used with suspected cervical spine injury.

Once youve utilized these techniques, reassess you patients breathing using the Look, Listen, and Feel
technique. Look for foreign bodies in the mouth and for adequate rise and fall of the chest, listen for air
movement at your patients mouth and feel for air movement on your face from the patients breathing.

I - 41
Take Home Message
Inadequate ventilation can be due to poor respiratory effort and/or airway obstruction.
Gurgling respirations can indicate foreign material in the airway. Consider suctioning the airway to
clear this.
Unconscious patients often obstruct their airway when their tongue falls into their posterior pharynx.
If a patient is not breathing (apneic) or has an obstructed airway, use the chin lift and/or jaw thrust to
open the airway.
Once youve opened the airway, assess ventilation with the Look, Listen, Feel technique.

Positive Pressure Ventilation


If establishing a patent airway does not restore a patients ventilation, then you must breathe for the patient. In
most situations this is accomplished by bag-valve mask (BVM) ventilation. Basically this technique uses a bag
and a face mask to push oxygen rich gas into the patients lungs.

Photo by: Dr. Ian Rigby used with permission

To do this, begin by setting up your equipment. Attach your BVM unit to an oxygen supply turned on to 15L per
minute. Ensure your bag units reservoir fills with oxygen. Place an appropriate size facemask on your bag unit
(the mask should cover your patients nose and mouth snugly).

Your left hand is used to apply the mask to the patient. Your first and second fingers shaped in a C
configuration are used to hold the mask over the patients nose and mouth creating an airtight seal. The third
and fourth fingers are used to lift the chin upwards (a type of chin lift). Finally your fifth finger hooks around the
angle of the mandible to provide a jaw thrust. So the hand is shaped in a C (first and second fingers) and E
(third, fourth & fifth) fingers.

With the neck extended and the jaw thrust forward, use your right hand to squeeze the bag, forcing oxygen into
your patients lungs. Pay particular attention to obvious air leaks around the mask. This will decrease the
effectiveness of your BVM ventilation.

Provide 10-12 breaths per minute for most patients who are not breathing. Watch for rise and fall of the chest
to indicate successful ventilation.
We should have some still photos of one and two person BVM will have to do this for the future version

I - 42
Arrrgh, I Cant Ventilate: Two Person BVM
BVM ventilation can be a challenging procedure. This is particularly true for those with facial deformities,
trauma or beards. Most often this is due to an inability to form a seal between the patients face and the mask.
In such circumstances, consider utilizing a two person BVM technique.

In the two person BVM technique, the first person uses both hands to form a seal with the mask and to lift the
patients chin. The second person is dedicated to ventilating with the bag.

Take Home Message


If opening your patients airway does not restore their ventilation, you need to breath for them.
The BVM technique allows you to force oxygen rich gas into your patients lungs.
In this technique your left hand stabilizes the mask on the patients face and provides a jaw thrust/chin
lift. Your right hand ventilates the patient by squeezing the bag.
This can be a difficult procedure. If you are having trouble ventilating the patient, consider using a two
person technique.

Airway Adjuncts place this before Positive pressure ventilation


One of the most utilized adjuncts for noninvasive ventilation is the oropharyngeal airway. This is a plastic
device that is inserted into the patients oropharynx and serves to keep the tongue away from the posterior
pharynx. Although this is a very useful tool, it should not be used in conscious patients as it can induce gagging
and vomiting.

If your patient is unconscious, it is preferable to insert an oropharyngeal airway to improve BVM ventilation.
Begin by selecting the correct size oropharyngeal airway. A correctly sized airway should go from the lips to the
angle of the mandible.

To place the oropharyngeal airway, begin by inserting it with the curve pointing towards the top of the patients
head (180 degrees from the way it will eventually end up). As you insert the airway, rotate it 180 degrees.
Once it is correctly positioned, the airway will end up with the curve pointing downwards and the distal tip of the
airway lying in the posterior pharynx
.It is becoming more acceptable in adults to place the oropharyngeal airway with the tip towards the glottis as
employed in pediatric airway insertion. With this in place, continue to ventilate the patient as you normally
would.

Please review the online video module available on the Osler podcast blog or Core site,
which demonstrates the steps discussed here, and then take a few minutes to complete
the attached open-book quiz prior to your small group session. Please bring your quiz
with you and submit it to your preceptor.

I - 43
Module #4 - Basic Airway Management

Open Book Quiz bring completed quiz to your small group session.
1. The most common cause of airway obstruction in an unconscious patient is due to

A) Vomitus in the airway


B) The tongue falling into the posterior pharynx
C) Lack of muscular tone of the epiglottis
D) Failure to clear airway secretions

2. Which of the following techniques does NOT prevent the tongue from obstructing the posterior oropharynx?

A) Jaw thrust
B) Placement of an oropharyngeal airway
C) Use of crichoid pressure
D) Chin lift

3. What is the correct ventilation rate for most patients?

A) 6 breaths per minute


B) 10-12 breaths per minute
C) 18-20 breaths per minute
D) 30 breaths per minute

4. An oropharyngeal airway is useful in ventilating

A) An unconscious patient
B) A combative patient
C) A patient with a foreign body in their airway
D) A patient with a beard

5. When ventilating a patient with bag valve mask ventilation, your left fifth finger performs which maneuver?

A) Jaw thrust
B) Chin lift
C) Neck stabilization
D) Squeezes the bag

I - 44
Procedural Skills Module #5
Advanced Airway Management
Introduction:
This is the written material to accompany the on-line video and small group skills session. It assumes that you
have done module #4 Basic Airway Management. This module focuses on the skill of endotracheal
intubation. This is a skill that is used in a variety of clinical settings. The most common is the OR, as part of
anesthesia. It is also frequently required in the emergency setting, either on the ward or in the emergency
department as well as the intensive care setting. We will be briefly touching upon medication usage as an
adjunct to intubation, but this is not a module on anesthesia induction. Airway management is also very
complex, and further modules, which you might be exposed to during residency, will build on the skills learned
here and expose you to the skills associated with the difficult airway, the pediatric airway and surgical
management of the airway.

Objectives:
Upon completion of this module, students will be able to do the following independently:
Identify the indications for intubation and positive pressure ventilation (PPV)
Describe the indications and contraindications of Laryngeal Mask Airway use
Describe the steps in Laryngeal Mask insertion
Correctly demonstrate Laryngeal Mask insertion on mannequin
Describe an appropriate approach to pre-intubation airway assessment
Describe the most significant risks and common complications of intubation and PPV
Describe the steps involved in endotracheal intubation
Correctly demonstrate endotracheal intubation on a mannequin
Demonstrate appropriate assessment of ETT placement
Describe the implications of various specific clinical situations on the need to intubate and the process
of performing an intubation
Display some awareness of various pharmacological adjuncts to endotracheal intubation

Part I The Need to Intubate

Indications
There are three clearly defined indications to perform endotracheal intubation. They are:
Failure to oxygenate
Failure to ventilate
Failure to protect the airway
Lets look at each of these individually.
Failure of Oxygenate
Failure to oxygenate occurs when a patient is unable to achieve adequate oxygenation through the use of
supplemental oxygen applied by facemask. It is usually considered to exist in patients whose SaO2 is less than
90% despite maximal oxygen delivery. Remember that once intubated, oxygen delivery will be slightly better
than the best facemask, as we can deliver 100% directly to the lungs. The reasons why a patient might be
experiencing failure to oxygenate are varied. The most common would be problems with the lungs (asthma,
COPD), problems with the heart (CHF), obstruction (foreign body aspiration) or failure to make spontaneous
respiratory effort (coma). If you stop to think about these various conditions, it should soon become obvious
that simply placing a tube into the trachea is not going to solve all these problems, and in the case of a foreign
body aspiration, might actually make things worse.

I - 45
Failure to Ventilate
Failure to ventilate is a related phenomenon, but not quite the same thing. In a patient who is experiencing
respiratory distress, poor gas exchange is manifested as both hypoxemia and hypercarbia. In certain
conditions (most notably COPD) a patient may be able to oxygenate adequately, often through passive
oxygenation, but as the CO2 level increases, metabolic derangements occur. Patients become acidotic and
obtunded. If they are unable to adequately blow off their CO2, intubation sometimes allows the physician an
opportunity to regulate their respiratory function and correct those abnormalities.

Another way to approach this is to think of patients in terms of cant breathe and wont breathe Cant breathe
patients are unable to maintain adequate gas exchange despite maximum effort, and need supplemental
assistance, either in the form of more oxygen or more forceful delivery. Wont breath patients are unable to
maintain adequate gas exchange due to failure to produce appropriate respiratory effort. This can occur for
metabolic, toxicologic or traumatic reasons.

Failure to Protect the Airway


The third indication for intubation is failure to protect ones airway. These patients generally fall into two groups
those whose level of consciousness is such that they no longer have an adequate gag reflex and are at risk of
aspiration, and those who have an airway obstruction. The former group is (thankfully) the most common, and
includes patients with significant CNS pathology (intracranial hemorrhage, meningitis) and patients who are
intoxicated or profoundly metabolically unwell (alcohol intoxication, sepsis). This group also includes patients
who are sedated whether from a general anesthetic or some kind of toxic ingestion. The later group are at
much more difficult to manage. These are patients who are at risk for airway occlusion due to some (usually)
evolving process, and in whom the placement of an ETT prior to complete airway occlusion is lifesaving.
Examples of this would be anaphylaxis / angioedema, airway burns, neck trauma, or an expanding neck mass.

Other Considerations
In addition to the three very well established indications mentioned above, there are other indications. One is if
the patients anticipated clinical course will eventually warrant an intubation. Various serious medical conditions
including shock, multi -trauma, congestive heart failure and deteriorating COPD may not respond to
conservative therapy and may warrant early intubation. This pre-emptive intervention will optimize care and
allow intubation in a more controlled setting before the patient approaches a full blown cardio-respiratory arrest.
Intubation is also considered to facilitate diagnostic imaging in combative patients and facilitate management of
patients who need ground or air transport to higher level facilities
Contra-indications
Given that endotracheal intubation is a potentially life saving intervention in patients who require it, there are no
true absolute contra-indications to the procedure. The only exception to this occurs in the setting of an
untreated pneumothorax where switching the patient from negative pressure ventilation to positive pressure
ventilation is going to make them worse, not better. There are relative contraindications to certain intubation
techniques and to various medications that are used to facilitate intubation and these will be touched on below.
A detailed discussion of these contraindications is beyond the scope of this teaching module.
Although it is not technically a contra-indication, it is always important to confirm with the patient or surrogate
decision maker that intubation is desired. Many patients who have expressed the desire not to be resuscitated
in the event of a significant life-threatening illness have specifically indicated that they do not want to be
intubated.
Take Home Message
Patients require intubation for three reasons: failure to oxygenate, failure to ventilate and failure to
protect their airways
Certain medications can be given via the ETT
Patients may require intubation before they satisfy one of the indications if it becomes clear that
they will do so imminently.
Always obtain informed consent for the procedure to the best of your ability

I - 46
Part II Getting Ready

Equipment
The first step of preparing for intubation (or any other procedure) is ensuring that you have at hand all the
necessary equipment. For intubation, you will require the
following:
An endotracheal tube: size 8 for most men, size 7.5 for
most women.
A 10 cc syringe to inflate the cuff on the ETT
A stylet: this is used to help guide the tube and provides
some rigidity
Lubricating jelly: to lubricate the end of the tube
A laryngoscope: there are several types of blades, the
most common being the Macintosh curved blade
Suction
Securing tie / tape to secure endotracheal tube to patient
A ventilator or bagging apparatus to attach the tube to.
A rescue device such as larygneal mask (LMA ) or bougie

Figure 2: ETT Examples & a stylet

It is, of course important to ensure that all the equipment is ready and in good working order. Having a
selection of different sized ETTs is also helpful.
Laryngoscope blades come in a variety of different shapes and
sizes. The most commonly used are the MacIntosh blade, which
is curved and is usually already on the laryngoscope in most
hospitals. They come in different sizes, the most common being a
4, which is generally sufficient for most adults. You will hear
people refer to them as Mac 4s or Mac 5s. The next most
common is the Miller blade, which is generally longer and straight.
Lastly there are also Wisconsin and Oxford blades. You will be
learning on a MacIntosh and Miller blade.

In addition to the above, you should also have available all the Figure 3 - Different Laryngoscope Blades
equipment which has been in used prior to intubation, and which you
learned about in the basic airway module, including facemask and bag valve mask apparatus.

Assessing the Airway


Prior to attempting to intubate someone, it is nice to have some idea of how difficult this might be to achieve.
Looking for certain features in your patient that predict difficult intubation can be helpful. Such features include,
but are not limited to:
Short neck
Small jaw
Poor jaw opening
Poor C-spine mobility
Big tongue
Prior surgery to the jaw/neck/mouth

Another feature which has been shown to be related to difficulty of intubation is what is called the Mallampati
Class. The details are not important at this point, but are described in the image below. For the time being, it is

I - 47
sufficient to notice that the less of the pharynx you can see when the patient opens his or her mouth, the more
difficult you can expect the intubation to be.

Source: Walls, R.M., Murphy, M.; Manual of Emergency Airway Management 2nd Ed., p78

Preparation
After all your equipment is ready, the next step is to prepare your patient for intubation. This amounts to proper
positioning and maximal pre-oxygenation. In most situations, you are going to want the patient supine on a
stretcher or table. Ensure that the table is raised sufficiently that you can perform the procedure without hurting
your back. Make sure you have all the people nearby that you will need for assistance. This includes
respiratory therapists, nurses, and at least initially, a more experienced physician. Preparation also means
having all your equipment available including suction, stylet alternate ET tube sizes, various sizes of
laryngoscope blades and potential back up or airway rescue devices including laryngeal mask or lighted stylet.
The second element of preparation consists of pre-oxygenation. It is truly remarkable how much oxygen can be
stored in the lungs and circulation of a patient who is allowed to maximally pre-oxygenate for several minutes
prior to intubation. In healthy young patients 3 minutes of pre-oxygenation with 100% O2 can provide sufficient
stores to allow several minutes of apnea prior to desaturation. This margin of safety is smaller in the elderly
and the very young, as well as in those who have underlying lung disease or large chest walls (i.e. obese
patients) and large abdomens (pregnant patients).
You will also want to prepare your endotracheal tube. With the stylet in place, bend the distal 2-3 cm at an
approximately 60 degree angle (this is referred to as the hockey stick). Next, you will want to apply some
lubricating jelly to the end of the tube to ensure that it is able to pass the cords as easily as possible.

Pharmacologic Adjuncts
Laryngoscopy and endotracheal intubation are incredibly noxious stimuli for any awake patients. Accordingly,
sedation with an induction agent is required for all but the most profoundly comatose. This is often
supplemented with neuromuscular blockade resulting in paralysis. Detailed discussion of these agents is well
beyond the scope of this module, but it is worth being aware of some of the medications you will see being
used. These include, but are not limited to:
Midazolam (a benzodiazepine sedative)
Fentanyl (a short acting narcotic)
Etomidate (an IV anaesthetic agent)
Isoflurane (an inhalational general anaesthetic agent)
Propofol (a short acting sedative agent)

I - 48
Ketamine (a dissociative anasthetic agent) put ketamine first followed by propofol then midazolam
Succinylcholine (a depolarizing neuromuscular blocking agent)
Rocuronium or Pancuronium (non-depolarizing neuromuscular blocking agents)

This module will not focus on the issues specific to managing anaesthetized and / or paralyzed patients.
Suffice it to stay that this adds further levels of complexity to the procedure in question. For the time being, it is
sufficient to be aware that these issues exist and require active management. You will learn more about this
during your anaesthesia clerkship, but feel free to ask questions of your small group preceptors who are all
expert in the area of airway management.

Take Home Message


Most adults are intubated using a 7.5 or 8 endotracheal tube, although other sizes exist.
Several different laryngoscope blades exist. The most common is the curved MacIntosh
blade
Pre-oxygenation prior to attempting intubation is very important
Several medications are used to facilitate intubation. The most common groups are
sedatives and paralytic agents

Part III Securing the Airway

Step 1 Position the Patient


As with many of the skills that you have already learned, such as
lumbar puncture, proper positioning is key to successful
completion of this procedure. What you want is to have your
patient in a position that will create a direct line of sight from the
mouth to the vocal cords. If the neck is overly flexed or
extended, this will not be possible. The optimal position for the
patient is with the neck flexed forward and the head tilted slightly
back. This is described as the sniffing position and can often Figure 4 - Sniffing Position
be facilitated by placing a towel or small pillow under the
patients head. Some people find that it is easiest to achieve by cupping the patients head in the right hand and
moving it into the appropriate position. Look at Figure 4 for a graphic representation of the profound difference
that proper positioning makes. Unfortunately in patients with suspected c spine injuries this position is
contraindicated.

Step 2 Laryngoscopy
Hold the laryngoscope in your left hand, and insert the blade into the patients mouth from the right side (for
once, your right and the patients right are the same thing!). As you bring the laryngoscope over to the midline,
use the blade to gently sweep the tongue out of the way. Next you want to advance the curved blade until the
tip is in the vallecula (the space at the base of the tongue). This will allow you to lift the epiglottis out of the way
so that you can see the cords. The blade of a Miller (straight) laryngoscope when placed correctly lifts the
epiglottis out of the way and is often more useful in pediatric patients and airways that are judged to be more
anterior.

The next step is to extend your left arm to lift the patients lower jaw and create some room to see what you are
doing. The really important thing here is that you are lifting the up and away from yourself. Hold the
laryngoscope so that the blade is at a 45 degree angle from perpendicular and move it as though it were
traveling along a guidewire running through its long access. This is very important for two reasons. First, it will

I - 49
help bring the airway into alignment allowing you a clear line of sight to the cords. It will also prevent you from
levering the laryngoscope back on the patients upper teeth (which often leads to chipped and broken teeth and
in the United States lots of lawsuits).

Step 3 Visualize the Cords & Pass the Tube


If the above has been correctly, you should be able to see the cords. What you see should look something like
this:

Source: Walls, R.M., Murphy, M.; Manual of Emergency Airway Management 2nd Ed., p39

If it does not, withdraw the laryngoscope and start again. The most common mistakes made by novice
intubators is that they tend to put the blade of the laryngoscope in too far and in so doing, push the epiglottis
down over the glottis, or that they tend to be off of midline. Ensure that you are in the midline, and try putting
the blade in a little less far.

If that does not work, you can try the BURP maneuver. This consists of having someone else place their
thumb and forefinger on the larynx and gently push down towards the bed, up towards the head and slightly to
the right - hence the name Backwards, Upwards, Rightwards Pressure.

Once you see the cords, take the endotracheal tube in your right hand and approaching the cords from the
right, slide it between them. Advance it 3 or 4 centimeters then inflate the balloon with air. Whenever possible,
you should attempt to directly visualize the tube passing between the vocal cords, but occasionally you will not
be able to do so. This is referred to as a blind intubation.

Step 4 Secure the Tube


The tube should be secured firmly in place. This will prevent the tube
from coming accidentally dislodged, and prevent self-extubation by a
patient who might be awake enough to know that she does not like
having this thing in her throat, but not awake enough to realize that she
needs it. This can be done by tying it in place with fabric or by means of a
commercially available device that holds it in place using a C-clamp.
In a pinch it can even be held in place by tape. You will likely also want
to use a bite-block, as patients can sometimes bite through their tube if
they wake up unexpectedly. Photo by: Dr. Ian Rigby used with
permission

I - 50
Take Home Message
Place the patient in the sniffing position to ensure your maximum chance of successful
laryngoscopy.
Use the laryngoscope to pull the lower jaw up and away from yourself. Never lever the
laryngoscope on the patients upper teeth
If you cant see the cords, try pulling the laryngoscope back slightly and try again.
Pass the tube through the cords under direct visualization whenever possible
Always secure that tube that you have worked so hard to place. Make sure someone
keeps a hand on it until it is tied on otherwise securely in place.

Part IV Confirming Successful Intubation


Confirming that the endotracheal tube is actually in the trachea and not in the esophagus is crucial, as patients
with unrecognized esophageal intubation tend not to do well. Direct visualization of the tube passing through
the cords is a great start, but is not sufficient to confirm tube placement. A number of other methods can be
use.
Auscultation over both lungs and epigastrium: Breath sounds should be clearly heard over both lungs.
It is also helpful to do this in order to determine the likelihood that the ETT has been pushed in too far
and is actually located in the right mainstem bronchus. In conjunction with this, auscultation over the
epigastrium should yield no sound of air being pumped into the stomach.

Esophageal detection device: These are small self-inflating bulbs that attach to the end
of the endotracheal tube. They are used by squeezing them, then attaching them to the
end of the tube. If they rapidly expand back to their original shape, that suggests that the
tube is located is in the trachea which will stay open in response to the venturi effect
exerted by the bulb, while the highly collapsible esophagus will occlude the tube and
prevent the bulb from re-expanding.
Misting in the tube: Look for misting in the tube on expiration. This is taken to
suggest that the tube is in the trachea, although is not 100% reliable. Photo by: Dr. Ian Rigby used
with permission

End-tidal CO2 Detector: This colorimetric detector is attached to the endotracheal tube and changes to
yellow or tan color if adequate carbon dioxide is detected but is less accurate in circulatory arrest.
Purple color signifies inadequate carbon dioxide hence purple = problem.
Capnography using infrared detection of carbon dioxide is a very reliable method of confirming proper
endotracheal tube placement
Generally, two methods of confirmation should be used.
Take Home Message
Direct visualization of the tube passing through the cords is great, but not sufficient to
confirm placement.
Ideally, two or more methods of confirming tube placement should be used.
The most reliable method of confirming tube placement in the trachea is through the use of a
capnography
Laryngeal mask airway although not a definitive airway can serve as a temporary rescue
airway

I - 51
Laryngeal Mask Airway (LMA)

Image courtesy of LMA North America

Whilst NOT a definitive airway, the LMA is a good alternative to prolonged BVM and as a rescue airway. It is
also referred to as a supraglottic airway. It is easy to insert with a higher success rate than intubation for
learners and lowers the risk of aspiration as compared to BVM. The distal portion of the LMA is designed to
cover the structures ABOVE the glottis. The proximal end looks very similar to that of an endotracheal tube.
There are many types of LMAs, but for convenience, there are Classic (reusable), Unique (disposable) and
Intubating LMAs (ILMAs), which facilitate intubation THROUGH the LMA.
Indications:
Alternate airway for short operative procedures
Difficult airway/rescue device
Cardiac arrest
To assist intubation (ILMA)
Pre-hospital airway

Contraindications:
Complete Airway Obstruction
Inability to open mouth
Increased risk of aspiration (such as actively vomiting)
Abnormal anatomy
Need for high degree of positive pressure ventilation
As with Intubation, the same equipment is
required (personal protection, suction, IVs, etc).
For placement of the LMA first the appropriate
size must be chosen which is weight based and
ranges from size 1 < 5kg to size 6 > 100kg. The
LMA cuff is then completely deflated with
application of lubrication on the posterior surface
of the LMA. The sniffing position is the best
position for the patient prior to insertion. The most
common method for insertion is accomplished by
holding the LMA like a pen with the index finger at
the junction of the airway tube and the cuff. Use
the index finger to slide the LMA along the hard
palate and into the hypopharynx until resistance
is felt. After the LMA is fully inserted inflate the
cuff enough to achieve a good seal with the
glottis. Often only half the maximum cuff volume
is required.

I - 52
Troubleshooting inadequate LMA ventilation includes modifying cuff inflation, head and neck positioning ( if no c
spine injury) withdrawing , advancing or rotating LMA or ultimately replacing LMA with a larger size which is
more likely required than to decrease size.
The supraglottic airways including LMA require less training to insert and may become first choice for
advanced airways but do have potential higher risk of aspiration especially in patients who have not fasted

Part V What If: Complications and Special Circumstances


Complications
Like any procedure in medicine, endotracheal intubation carries with it several risks. These risks often pale by
comparison to the dire situation which has lead to intubation being considered in the first place. That having
been said, many of these risks can be minimized by careful performance of the procedure.
The complications of endotracheal intubation can be classified into four groups: immediate complications
related to laryngoscopy and tube placement, delayed complications related to an indwelling ETT, complications
related to the use of positive pressure ventilation (PPV), and complications related to the use of adjuvant
medications.

Immediate Complications
Chipped Teeth. This is usually the result of poor laryngoscopy technique, and your goal should be to
never chip a tooth.
Pharyngeal Swelling and Bleeding. If difficulty in visualizing the cords occurs, and multiple attempts at
laryngoscopy and at blindly passing the tube are made, significant trauma to the airway can ensue.
This can be important in a patient with an already compromised upper airway secondary to burns,
trauma or edema.
Raised Intracranial Pressure. The noxious nature of laryngoscopy can induce a significant but transient
rise in ICP. This is generally not clinically relevant, except in the setting of patients with known or
suspected intracranial lesions.
Bradycardia. Laryngoscopy can also induce a vagal response. This is most significant in children
where it can be profound enough to induce a bradysystolic arrest. With pediatric patients, most
practitioners attempt to address this complication with a prophylactic dose of atropine prior to
intubation.
Vocal Cord Avulsion. Overly forceful attempts to pass an endotracheal tube past closed or partially
closed vocal cords can result in significant trauma to the cords themselves. In the worst case, the
cords themselves can be avulsed, requiring subsequent surgical repair.
Delayed Complications
Subglotic Stenosis. In some cases, prolonged placement of an ETT results in pressure damage to the
trachea for the inflated cuff. This is usually a problem in the ICU setting only.
Tube Migration. On occasion, an endotracheal tube may become dislodged or migrate more distally
(i.e. into the right mainstem bronchus). Patients are at higher risk for this when they have frequent
manipulation of the tube or are moved often. It is a complication that should be considered when a
previously stable intubated patient suddenly becomes unstable. You cannot assume that because you
put the tube in the right place that it will necessarily stay there.
Tube Obstruction. During your hands on session, take a careful look at the internal diameter of a 7.5
ETT. After doing so, you will appreciate how easy it is for thick secretions to obstruct a tube. Usually
these can be cleared with suctioning.
Complications Related to PPV
Barotrauma. PPV is usually provided at the lowest possible airway pressures. When a patient is
breathing spontaneously, however, those pressures are in fact negative. In patients who are ventilated
at overly high volumes or rates, damage to the lung tissue can ensue and can in some cases be quite

I - 53
severe. This is also a significant issue in patients with obstructive lung disease in whom high
ventilatory pressures are necessary in order to get sufficient oxygen delivery to the alveoli.
Pneumothorax. This is really a form of barotrauma, but requires immediate decompression in the
setting of PPV. Unlike the case of someone who is breathing spontaneously, a pneumothorax in a
patient receiving PPV will progress rapidly to tension pneumothorax. In the spontaneously breathing
patient, the pneumothorax grows until the pressure in the pneumothorax is equal to the inspiratory
pressure created by breathing. The pressures applied in PPV are however much greater, and with
each subsequent breath, more air is forced into the pneumothorax thus causing the lung to collapse
even more.
Decreased Cardiac Venous Return. One of the biggest differences between the spontaneously
breathing patient and the patient receiving PPV is in regards to the impact respiration has on venous
return to the heart. In the breathing patient the negative intrathoracic pressure that is generated acts to
draw blood from the vena cava into the chest and subsequently back to the heart. Contrast that to the
setting of PPV, where high intrathoracic pressures actually impede venous return. Remember that
many of the people being intubated for respiratory reasons have been struggling to breath, and as such
have been generating very high negative intrathoracic pressures. When these patients are then
intubated, their intra-thoracic pressure skyrockets. This can result in a precipitous decline in their
venous return and, in some cases, overt circulatory collapse. Many clinicians attempt to prevent this by
giving fluid boluses prior to intubation, or small doses of vasoconstrictive medications at the time of
intubation.

Complications Related to Medication Use


The medications used as adjuncts to endotracheal intubation have many and varied side effects. You
will learn about these in great detail over the next few years. They vary from the trivial to the
immediately life threatening.
The most feared complication of medication use in this setting is the development of what is described
as the Cant intubate, cant ventilate scenario. This describes the setting in which a patient has been
sedated and paralyzed, and then you find yourself unable to intubate the patient. You stop trying, but
then also find that you are unable to perform bag-valve-mask ventilation. In short, you have taken
away the patients ability to breath for himself, and are unable to breath for him. This is where back-up
comes in, as well as specific difficult airway skills which you will learn later.

It is important to keep all these potential complications in perspective. Keep in mind that failing to intubate a
patient who requires it is almost universally leads to the death of the patient, which makes a chipped tooth or
even an avulsed vocal cord seem fairly trivial by comparison.

Special Circumstances
While there are lots of subtleties to intubating various groups of patients, three specific populations are worth
mentioning here.

(1) COPDers and Asthmatics: These patients are challenging, as mentioned above, they can suffer quite
deleterious effects as a result of intubation. An asthmatic will be breathing very quickly prior to intubation. This
keeps their CO2 relatively low until they begin to get tired at which time it begins to rise. Prolonged attempts at
intubation will lead to a rapid increase in their already elevated CO2, which will rapidly increase acidosis and
can lead to cardiovascular collapse. The other issue in the long run is that their problem is getting air out, not
getting it in. These patients already suffer from airway obstruction and gas trapping. It seems counter intuitive
then, to intubate them and start forcing more air into their already over-expanded lungs. Remember that the
point of intubating these people is to take over their work of breathing because they are nearing or have
reached the point of exhaustion. Once you are breathing for them, you want to slow their rate down and

I - 54
provide plenty of time for exhalation (short inspiration, long expiration). Ultimately their CO2 will rise again but
hopefully in a controlled fashion and not to the point of circulatory collapse. This is known as permissive
hypercapnea.

(2) Patients with Raised Intracranial Pressure: These patients often require intubation because they wont
breathe spontaneously and yet the entire process of laryngoscopy and intubation is very noxious to them and
invariably raises their ICP further. This can be potentiated through the use of certain pharmacologic agents
(lidocaine, fentanynl, thiopental), but is otherwise a necessary evil.

(3) Patients with Possible Foreign Body Aspiration: In a patient who might have a partial airway obstruction due
to a foreign body, you want to think twice before you go charging towards intubation. They will certainly look
like they really need help to breathe. The problem is this: Imagine that they have something sitting just above
the cords (say a broken off tooth) which is partially blocking their airway every time they breathe in. You then
stick the laryngoscope in their airway, and before you can see where it has gone, you might well push that
foreign body the rest of the way into the airway, giving them a complete obstruction.

Take Home Message


Complications are common, but generally outweighed by the need for intubation in the
first place.
Knowing what complications to expect will help you manage the ones that can be
managed when they occur.
Always consider the impact that switching a patient from negative pressure to positive
pressure ventilation is going to have on their physiology.
Consider the special circumstances of patients with raised ICP, COPD, or possible
foreign body aspiration prior to proceeding with intubation.

Please review the online video module available at on the Osler podcast blog or Core
site, which demonstrates the steps discussed here, and then take a few minutes to
complete the attached open-book quiz prior to your small group session. Please bring
your quiz with you and submit it to your preceptor.

I - 55
Module #5 Advanced Airway Management
Open Book Quiz bring completed quiz to your small group session.

1. Which of the following is NOT an indication for intubation?

(a) Failure of ventilation


(b) Failure of oxygenation
(c) Failure to protect the airway
(d) Failure of exhalation

2. What type of laryngoscope blade is curved?

(a) MacIntosh
(b) Miller
(c) Wisconsin
(d) Bronchoscope

3. What characteristic(s) below are risks for a difficult airway?

(a) Poor neck mobility


(b) Large tongue
(c) Short neck
(d) All of the above

4. The tip of the MacIntosh laryngoscope blade should sit where?

(a) In the valeculla


(b) At the vocal cords
(c) Below the epiglottis
(d) Below the false cords

5. Which of the following is NOT a technique for confirming endotracheal tube placement?

(a) Auscultation of the chest and epigastrium


(b) End Tidal CO2 detection
(c) Esophageal detection device use
(d) Oscillatory oxygen measurement

6. Indications for laryngeal mask airway include all but which of the following?

(a) cardiac arrest


(b) rescue airway
(c) trauma airway
(d) brief operative procedures

I - 56
Procedural Skills Module #6
Casting and Splinting
Objectives:
Upon completion of this module, students will be able to do the following independently:
Determine indications for casting and splinting
Discuss complications of casting and splinting
Apply a short arm cast
Remove a short arm cast
Apply a volar splint, an ulnar gutter splint and a thumb spica splint

Immobilization with casting or splinting:


reduces pain and swelling
promotes bony alignment
provides protection
promotes healing of many conditions.

Casts and splints are routinely applied to various fractures but immobilization can also be used for tendon and
skin lacerations, soft tissue and joint infections, sprains and dislocations, as well as other inflammatory joint
conditions .

Immobilization has its hazards as well. As casting involves circumferential immobilization, complications are
more frequent with casting than splinting. Complications encountered with both casting and splinting include:
Dermatitis
Infection
Thermal Injury
Joint Stiffness
Neuropathies
Skin Breakdown
Ischemia
Compartment Syndrome

Although skin breakdown is probably the most common complication from a poorly applied cast, compartment
syndrome is the most dreaded complication. Compartment syndrome is rare and difficult to diagnose but it is
characterized by increasing pain often disproportionate to the injury. Other symptoms include parathesias,
paresis or paralysis and palpable tenseness of the compartment involved. Pulses are usually preserved until the
very late stages and definitive diagnoses are made with fasciotomy.

Casting
Casting is defined as the circumferential application of either plaster of paris or fiberglass to an injured
extremity. Casting is a more technical procedure, requires more time, provides better immobilization but has
higher complication rate than splinting. Plaster of paris is cheaper, is easier to mold but it takes longer to set, is
heavier and creates more mess than fiberglass.

I - 57
Equipment for short arm cast:
Stockinette for skin protection:
o 2 -3 inch for forearm
o 1 inch for thumb
Cotton Padding: 2 or 3 Inch for skin and bony prominence protection
Plaster or fiberglass: 2 or 3 inch
Bucket of cool or tepid water
Non sterile gloves
Gown
Towel
Cast scissors

Application of Short Arm Cast

The short arm cast is most commonly applied for distal radius and ulna injuries.

1. Clean and dry the arm, have the patient assume the position of function (hand holding a beverage can),
ensure patient comfort - consider putting the elbow on a table.

2. Cut 2 stockinette too long ~ antecubital crease to few cm past fingers.


3. Cut 1 stockinette for thumb about twice thumb length.
4. Put wider stockinette on forearm pull just proximal above antecubital fossa and place 1 stockinette on
thumb flatten out wrinkles.
5. Apply 2 or 3 cotton padding starting at wrist.
6. Keep roll close to skin overlap about 2/3 with each roll.
7. Do not cinch tightly just allow roll to unwind.
8. Keep padding just proximal to metacarpals distally and just proximal to antecubital fossa.
9. Apply one to two layers of padding and add extra to bony prominences as required.
10. Minimize wrinkles by tearing padding not cutting.
11. Cotton wrap with fifty percent overlap.

I - 58
Cotton wrap with fifty percent overlap:

12. Dip fiberglass or plaster in tepid water, NOT warm or hot and squeeze out extra moisture especially
with plaster. Apply casting material in similar fashion to padding material, overlapping approximately
fifty percent with previous wrap. Avoid wrapping too tightly, keep roll close to skin and minimize
wrinkling as this can lead to skin breakdown. When applying casting material in palmar region of hand
cut material rather than twist for more functional cast.

Short Arm Cast:

I - 59
13. Typically two layers of casting material are required for a proper cast. After the first layer of casting
material is laid down pull down the stockinette and cover with second layer. After applying the second
layer use the palmar surfaces of hands to smooth the casting material and stimulate the hardening of
the cast.

Avoid casting too distally over the metacarpals and too proximally into the antecubital fossa as both of these
errors leads to discomfort and diminished function. Be careful not to leave any sharp edges of casting material
with direct skin contact as this can be painful and lead to skin breakdown.

After casting material is applied and before the cast material has hardened, molding of the cast is required.
This involves using the palmar surface of the hands NOT fingertips. Modify the shape of the cast from a
cylinder to a flatter shape. This molding minimizes movement of injured extremity, maintains fracture reduction
and also helps the cast from becoming too loose. Three point molding is the term describing proper molding
technique and is pictured below.

I - 60
Cast Care
Once the cast is applied it needs to fully harden for several hours. The affected limb should be elevated to
reduce swelling and the cast should be kept clean and dry. If the cast gets wet it can lose its strength and there
can be subsequent skin maceration and breakdown.

Nothing should be inserted between the cast and skin as this will lead to skin irritation, laceration and/or
infection. If the cast appears too tight, it is often because the patient has not elevated the limb enough. If in
doubt the cast can be split in a longitudinal fashion with either one (univalve) or if necessary two (bivalve) cast
saw incisions.

Cast Removal
If a cast needs removal it is best accomplished either by cutting with cast scissors when it is still very soft and
has not cured or a cast saw is used when the cast is hard. The cast saw can cut and burn skin so BE
CAREFUL when using this tool. It is very noisy and frightening for pediatric patients. It should be used with
supervision initially. It is important to cut the cast using an up and down motion not a sliding or rolling motion.
When cutting through the cast material you will feel a give with the saw which signifies penetration through the
cast material into the padding --so go no further. After cutting the cast, spread the cast with cast spreaders, cut
through the padding with cast scissors (avoid cutting skin) and remove the cast.

Splinting
Immobilization of an affected body part using non circumferential application of plaster, fiberglass, metal or
other material.

Splinting has fewer complications than casting and is often employed for briefer periods of immobilization.
Splinting is not as effective for maintaining fracture reduction. It can be used for multiple areas of injury
including knee, shoulder, ankle, fingers. We will be focusing on application of volar forearm splint, ulnar gutter
splint and thumb spica splint.

Volar forearm splint


Typically used for a soft tissue injury to the hand such as: significant lacerations or burns, some carpal and
metacarpal fractures.

I - 61
Ulnar gutter splint
Routinely used for very common boxers fractures of the fifth metacarpal. It can be used for 3rd and 4th
metacarpal and phalangeal fractures as well as soft tissue injuries

Thumb spica splint


Typically used for soft tissue injuries/fractures of the thumb as well as possible scaphoid injuries. A scaphoid
cast is used for definitive scaphoid fractures.

Equipment for forearm splints:


Fiberglass 3-4 inch pre made splinting material or flat 4 inch sheets of plaster or fiberglass
Bucket of tepid water
Cotton padding (optional)
2 or 3 inch stockinette
Cast scissors
non sterile gloves
Elastic bandage material

Ulnar gutter splint:

Volar forearm splint:

I - 62
Thumb spica splint:

1. Apply stockinette to affected area to protect skin, making sure to cut sufficient length.
2. Consider applying cotton padding using technique described in casting
3. Measure splint and cut splint for appropriate size from approx mid or proximal 2/3 forearm to PIP or
DIP joints depending on injury and location
4. Immerse splint in water and squeeze out excess water- if pre-made splint apply padding to skin and
fiberglass away from skin.
5. Mold splint to affected part and wrap splint with elastic bandage. Hold in place till fiberglass or plaster
hardens and conforms with bone and soft tissue. Secure wrap with tape or fasteners.

Splint can be removed or adjusted as required by patient or caregiver by simply removing elastic wrap and
splint and rewrapping as needed.

Please review the online video module available at on the Osler podcast blog or Core
site, which demonstrates the steps discussed here, and then take a few minutes to
complete the attached open-book quiz prior to your small group session. Please bring
your quiz with you and submit it to your preceptor.

I - 63
Module #6 Casting and Splinting
Open Book Quiz bring completed quiz to your small group session.

1. List 4 indications for immobilization:

2. List 4 complications of immobilization

3. In a short arm cast what areas do you want exposed?

a) Carpal metacarpal joint


b) Metacarpal heads
c) Proximal phalanx thumb
d) Antecubital fossa

4. For dipping casting material prior to application, what temperature do you want the water?

a) Warm
b) Hot
c) Ice slurry
d) Tepid

5. When cutting a hardened cast what is the proper technique?

a) Use the saw to burn through the casting material


b) Use cast scissors to score the cast then use cast spreaders
c) Slide the saw back and forth across the cast until it is bivalved
d) Use the cast saw in an up and down motion to split the cast

I - 64
Casting and Splinting Quiz Cont

6. When applying a splint the proper sequence is:

a) Measure and cut stockinette, measure and cut splint material, apply splint fiberglass to skin, dip arm in
bucket , mold to injured part, elastic wrap splint
b) Measure and cut stockinette, measure and cut splint material, dip splint in water, squeeze out water,
apply splint cotton to skin, mold splint to injured part, elastic wrap
c) Measure and cut splint material, dip splint in water, squeeze out excess water, cotton to skin, mold to
injured part, elastic wrap
d) Measure and cut stockinette, dip splint in water squeeze excess water, cut to fit . mold to injured part
elastic wrap

7. When wrapping padding around arm be sure to:

a) Keep roll distant from arm to minimize contact with injured part
b) Minimize wrinkles by pulling tightly around bony prominences
c) Overlap about 2/3 with each wrap
d) Mold padding with a 3 point technique

8. When applying an ulnar gutter splint to affected limb ensure that:

a) The fiberglass or plaster portion of the splint is adjacent to the skin


b) The splint is tightly wrapped with elastic bandage
c) The padded portion of the splint is adjacent to the skin
d) The splint encompasses all four fingers leaving the thumb free

I - 65
Procedural Skills Module #7
Ultrasound Skills and Paracentesis

Introduction:
Welcome to the procedural skills Ultrasound (u/s) skills and paracentesis module. This module is comprised of
this article and a hands-on skills session. This article is to provide you with a basic understanding of
ultrasonography and its uses to facilitate procedures, specifically vessel identification and aspiration, and
paracentesis. Our goal is not to churn out a class of interventional radiologists, but to expose you to the
wonderful world ultrasound guided procedures. Have fun & get ready to get goo-ed!

Objectives:
Basic understanding of the principles of ultrasonography
Basic understanding ultrasound machines
Basic understanding of ultrasound guided procedures, specifically:
vessel identification and aspiration
paracentesis
Discuss the indications and contraindications for paracentesis

Ultrasound Basics:
Ultrasound is great for many reasons: portable, no radiation, quick, simple to use, but most importantly, it can
provide life altering diagnoses and facilitate procedures. Ultrasound is your friend. Learn to love it!

There are many terms associated with ultrasonography, which I will try and define SIMPLY for you. The actual
physics of ultrasonography are somewhat complex and I am both too lazy and stupid to describe them in detail
for you. There are many excellent War & Peace-thickness tomes written on this fascinating subject (snore!) that
you can find for yourself if you are really interested. It has something to do with the piezoelectric effect. Feel
free to casually drop this fact during the module to sound smart and impress your preceptors. If they ask you to
explain it, drop to the floor and fake a seizure. Sometimes its better to accept that something works as
opposed to banging your head against a wall trying to get it to stick in your brain.

I - 66
The principle of ultrasonography is based on waves at higher frequencies that can be heard by human ears,
hence ULTRAsound. The units of frequency are expressed in MegaHertz or MHz. Generally between 2-10
MHz in most probes, although some probes have even higher frequencies.

Here is how it works: Ultrasound waves are generated through a transducer (probe), they travel through or
PENETRATE a medium, during which time they lose some of their energy (ATTENUATION). The waves are
eventually REFLECTED back to the probe to create the image that you see on the machines screen. The
difference in IMPEDANCE between tissues (how much u/s waves can penetrate through a tissue) will provide
an INTERFACE so as to allow you to differentiate between 2 or more structures. In general: liquid has LESS
impedance so looks black, tissues/organs have more impedance so look less dark, bone has even more
impedance and reflects MOST u/s waves, so it looks much whiter.

The higher the FREQUENCY, the higher the RESOLUTION (sharpness/clarity of image) BUT the lesser the
PENETRATION. The lower the frequency, the lesser the RESOLUTION BUT the more PENETRATION. In
other words, higher frequencies make better pictures but you cant see as far into the structure. The lower the
frequency, the less sharp the image, but you can go deeper/farther into the structure being scanned.

What about the ultrasound gel or goo? The goo provides a medium through which the u/s waves can travel.
Goo is good. You can never have too much.

Here are some other terms that you need to know:

Near Field: the TOP half of the ultrasound screen and the part of the structure being scanned that is
CLOSEST to the probe.

bluephantom.com

I - 67
Far Field: the BOTTOM half of the screen and the part of the structure being scanned that is FARTHEST from
the probe.

bluephantom.com

Echogenic: something that appears WHITER onscreen because it reflects more u/s waves. (more echogenic =
whiter = HYPERECHOIC)

Echolucent: something that appears BLACKER onscreen because it reflects less u/s waves (they pass through
it). (more echolucent=blacker=HYPOECHOIC)

Isoechoic: the u/s waves travel though all structures onscreen equally so there is no interface (you cant see
anything)

Anechoic: NO reflection of u/s waves within a structure (looks black)

Scatter: u/s waves deflect off small and/or irregular structures at odd angles and prevent image generation.
Gas is the usual culprit. Gas is the enemy of ultrasound. It is evil. And it smells bad.

Got all that?

I - 68
Ultrasound Machines:

There are many different brands, types, makes of ultrasound machines, but most if not all will have certain
commonalities. Below is a summary of all you really have to know to get going.

Transducers (probes):

There are different MODE settings for transducers. For our purposes, we will be using B mode or brightness.
There is something called array when referring to probes. Throw that one in the piezoelectric effect pile.

There are different probes of all shapes, sizes and frequencies. The small, thin FLAT LINEAR ARRAY
PROBE, commonly known as the vascular probe is of high frequency.

I - 69
There is the PHASED ARRAY (SECTOR) PROBE, commonly known as the cardiac probe, of intermediate
frequency.

There is the CURVED LINEAR ARRAY (SECTOR) PROBE, commonly known as the abdominal probe, of low
frequency.

Finally, there is the ENDOCAVITARY (SECTOR) PROBE, of intermediate frequency. Not pictured, as we will
not be using this probe.

Linear format: square image onscreen, good resolution

bluephantom.com

I - 70
Sector format: pie shaped image onscreen

bluephantom.com

The important part of the probe is called the FOOTPRINT. It is the soft, usually greyish coloured area that
touches the patient.

sonosite.com

DO NOT DROP THE PROBE, ESPECIALLY ONTO THIS PART OR IT WILL BREAK THE PROBE!!!!

There is a marker on every probe. It is either a bump or dot or line on ONE side of the probe (Please refer to
above pictures of probes, marker shown by arrow). This marker should always point towards the PATIENTs
RIGHT side (TRANSVERSE view) or towards the PATIENTs HEAD (LONGITUDINAL view).

There are always exceptions to the rule (outlined later), but just go with me on this one.

I - 71
Sonoguide.com

When the probe is on the patient in the TRANSVERSE view, SCREEN LEFT will correspond to the PATIENTs
RIGHT side. In the LONGITUDINAL view, SCREEN LEFT will correspond to the PATIENTs HEAD.

The structure of interest (the thing that you are looking for with the ultrasound) should be centered on the
screen. In order to do this:

TRANSVERSE view:
To move the structure of interest to the RIGHT, move the probe towards the patients RIGHT
To move the structure of interest to the LEFT, move the probe towards the patients LEFT
LONGITUDINAL view:
To move the structure of interest to the RIGHT, move the probe towards the patients HEAD
To move the structure of interest to the LEFT, move the probe towards the patients FEET

***THIS WILL MAKE WAY MORE SENSE WHEN YOU ARE ACTUALLY
USING THE ULTRASOUND MACHINE***

I - 72
Probe movements:

Move SLOWLY so you dont miss what you are looking for.

SLIDING: moving the probe up and down, forward and backward along the skin to search out an area of
interest.

skin

SWEEPING: ANGLING the probe against the skin while keeping the probe contacting the skin in the SAME
position all of the time.

skin

I - 73
Ultrasound Machine Knobology:

There are a whole bunch of fancy buttons and knobs on most machines. They are nice to know about but there
are only a few practical ones for our purposes. Most machines have batteries, and also plugins. Some
machines have functions that will only work when plugged in, so best to do so. The ON/OFF button is always
helpful.

The GAIN dial (or buttons on some machines) allows for adjustment of the amount of waves returning to the
probe, thus increasing the sensitivity of the probe. This comes at a cost however. More gain=more scatter. The
MORE the gain, the WHITER the image will appear. The LESS gain, the DARKER the image will appear.

I - 74
DEPTH allows the machine to adjust from how deep the u/s waves will be displayed on the screen.
INCREASING depth shows MORE of the structure that you are scanning but things will look SMALLER on the
screen (LESS magnification). DECREASING depth shows LESS of the structure that you are scanning but
things will look BIGGER on the screen (MORE magnification).

Ideally you would like your area of interest (the thing that you want to see) to take up 2/3 of the screen and have
the object CENTRED on the screen.

Ultrasound Guided Procedures:


Vascular Identification and aspiration:

The whole point of this portion is get you accustomed to practicing using the ultrasound to assist you in a
procedure. Specifically using the ultrasound to help guide a needle into a structure. This will NOT be a manual
on ultrasound guided vascular access thats for another day.

As with any procedure, sterile technique is CRUCIAL. So cleansing the area, gowning, gloving, masking,
draping AND covering the ultrasound probe with a sterile probe cover are ESSENTIAL. On our block models,
we just want you to find the black circle and stick a needle in it.

To begin this procedure, use the VASCULAR PROBE to identify a vessel in a TRANSVERSE view.

It looks like a black circle that you can squish by applying pressure with the u/s probe. CENTRE the vessel
onscreen and adjust your depth so that it encompasses approximately 2/3 of the screen. Note the DEPTH of
the vessel by using the hash marks (little lines) on the side of the u/s screen. They will be in 0.25-1cm
increments, depending on your DEPTH setting. Holding the probe steady on the skin with your LEFT hand,
insert the needle (held in your RIGHT hand) at a 45 degree angle to the skin CLOSE to the CENTRE of the
probe. SEE the needle enter the skin ONSCREEN. You will be ASPIRATING/PULL BACK on the syringe with
your thumb whilst advancing the needle. SLIDE the probe in the same direction of the advancing needle so
that you can ALWAYS see THE TIP of the needle advancing towards your goal (middle of vessel/squishy black
circle). When you see the needle getting close to the vessel, the vessel will compress/squish somewhat. Next
you use the probe to watch a needle advance IN REAL TIME into/puncture that vessel.

I - 75
bluephantom.com

Once you puncture the vessel, you will feel resistance on the syringes plunger give way and a flash of blood
will fill the syringe (hopefully the dark, bluish venous kind, and not the red, pulsing arterial kind). You can turn
the probe 90 degrees to create a LONGITUDINAL view of the vessel to ensure that the needle is INSIDE the
vessel and not THROUGH the vessel entirely.

bluephantom.com

You will be aspirating on the needle as it is advancing, looking for flashback of blood to see if you hit your target
thats the other clue that youre in the right spot! In general, at an angle of insertion is 45 degrees to the skins
surface, the needle will have to travel 1.4 times the depth of the vessel to puncture the vessel wall.

45 degrees
Skin
1cm 1.4cm

vessel

I - 76
One important artifact that you will notice whilst performing vascular access is the RING DOWN ARTIFACT
caused by the needle. It will look like a series of parallel lines going down from the needle tip. This will make it
look as though the needle is deeper than it actually is. It is caused by vibration of the needle as the u/s waves
pass around it.

sonoguide.com

On a real person, generally we want to be accessing VEINS, not ARTERIES. It is important to know how to
figure out which is which. People do have anatomic variations that can make this difficult if we were to just go
on presumed anatomic location (hence u/ss use). Veins & arteries usually live close to each other. Under u/s,
veins will be THINNER WALLED, MORE easily COMPRESSIBLE, and will VARY with RESPIRATION. Veins
will often appear MORE PULSATILE than arteries so do not fall into that trap. Also, it is with ultrasound guided
vascular access that the probe marker rules are thrown out the window. For vascular access in the transverse
view, the probe marker always points to the OPERATORs (YOUR) LEFT side.

Now that you are a pro, lets move onto Paracentesis!

Paracentesis
Paracentesis is the removal of fluid from the peritoneal cavity with a needle. Indications for paracentesis can be
for diagnostic and/or therapeutic reasons.

Diagnostic indications for paracentesis:


1) analysis of new onset ascites (cancerous cells, transudate vs exudate, etc), 2) suspected bacterial
peritonitis

Therapeutic indications for paracentesis:


1) relief of intra-abdominal pressure caused by ascites
2) relief of respiratory distress caused by ascites.

I - 77
Contraindications for paracentesis are:
1) Patient refusal
2) Disseminated Intravascular Coagulation
3) Primary Fibrinolysis
4) Massive bowel distension
5) Proceeding through surgical scars (high incidence of bowel adhesion)

Coagulopathy is NOT an absolute contraindication for paracentesis.

Complications of paracentesis:
1) ascitic fluid leak (~5%, most common)
2) infection
3) hemorrhage (severe) esp if puncture the inferior epigastric artery
4) death (least common I should @*&$# hope so!!!)

Materials Needed:

Sterile gown, sterile gloves, mask, ultrasound machine & sterile probe covers, sterilizing solution, anaesthetic
solution, needles (for freezing: 27 gauge, varying sizes for the procedure: 1-3.5 inch, 14-22 gauge), syringes,
vacuum bottles, paracentesis/thoracentesis kit, fluid culture bottles (aerobic and anaerobic), cell count and
chemistry tubes, a documentation of consent.

Paracentesis/thoracentesis kit.

Paracentesis Procedure:
As with any procedure, sterile technique is CRUCIAL. So cleansing the area, gowning, gloving, masking,
draping AND covering the ultrasound probe with a sterile probe cover are ESSENTIAL.

There are two methods to perform ultrasound-guided paracentesis: the static and the dynamic. We will focus on
the dynamic approach.

I - 78
First we need to identify if our patient has ascites. This can be done either BEFORE or AFTER creating a sterile
field. Place the probe in the LONGITUDINAL view in either flank. Identify the kidney and place the inferior
portion of the kidney in the centre of the screen so we are FAR from the liver or spleen. Next move the probe in
all directions to find the LARGEST pocket of fluid.

bluephantom.com

This pocket is USUALLY in the flanks, but can be on the anterior abdomen. Take care to recognize structures
such as organs, bowel, bladder and vessels (esp Inferior Epigastric artery) so as to NOT puncture these
structures whilst performing the procedure.

Once this area is located, the static method involves marking the skin over this site and then proceeding
WITHOUT u/s guidance. The PREFERED method is the DYNAMIC method, during which u/s is used
CONTINUOUSLY to aspirate fluid AND to AVOID hitting the wrong things! So DONT do the Static. Keep it
Movin Baby!

If you have not done so already, open your paracentesis tray & gather all of your equipment. And create your
sterile field by cleansing the area with a cleaning solution then draping the area. Ensure that your u/s probe is
covered with a sterile cover. (An assistant must hold the probe for you, footprint up. Squeeze the packet of
sterile gel onto the probe footprint. Next stick your hand into the probe cover and grab the end like sticking
your hand inside of a sock to make it a sock puppet. Grab the probe with your hand whilst inside the probe
cover, then pull the rest of the cover down around the probe. Secure the cover with an elastic.

Relocate the largest pocket of fluid and then using our already learned u/s guided aspiration skills, advance the
needle into the collection of fluid and aspirate your fluid. The peritoneum is very pliable and will often tent
down a few centimeters whilst trying to puncture the peritoneal cavity. After aspiration, remove the needle,
cover the wound with a dressing and discard your sharps.

I - 79
bluephantom.com

I will mention the Z-track technique, which can be used in the blind and static u/s approaches. Thats all youre
going to hear about it, because it is not used in the dynamic approach.

Remove the fluid and place it in the various tubes from the lab for fluid analysis. The analysis of peritoneal fluid
is not the purpose of this module and will not be discussed.

For Therapeutic paracentesis a LARGE volume of fluid is to be removed for symptom control. A large volume
paracentesis is the removal of greater than 5 L of fluid. The procedure is similar except you will use a larger
gauge angiocath (like an IV sharp metal centre/trocar with a blunt plastic sheath overlying) usually 14-18
gauge. Because there is a larger volume of fluid to be removed, aspirating via syringe is impractical and time
consuming (especially for emergency physicians who will lose interest after about 5 seconds and will seek out
some other stimulus, like that shiny balloon over there, hey everybody look at that balloon! Now look at that
fancy machine that makes a beeping sound! How cool! I digress). After aspirating fluid and confirming you
are in the peritoneal cavity (with our friend Mr. Ultrasound), the CATHETER can be advanced OVER the needle
and then connected to a vacuum container, which will syphon out the fluid by means of negative pressure.

Please review the online video module available at on the Osler podcast blog or Core
site, which demonstrates the steps discussed here, and then take a few minutes to
complete the attached open-book quiz prior to your small group session. Please bring
your quiz with you and submit it to your preceptor.

I - 80
Module #7 Ultrasound Skills and Peracentesis
Open Book Quiz bring completed quiz to your small group session.

1. Which statement is TRUE:


a) HIGHER Frequency provides MORE Penetration
b) LOWER Frequency provides BETTER Resolution
c) LOWER Resolution is due to LOWER Penetration
d) HIGHER Frequency provides LESS Penetration

2. Which Statement is FALSE:


a) Paracentesis can be done for diagnostic and therapeutic reasons
b) Contraindications for paracentesis include patient refusal
c) An isolated elevation of INR is NOT an absolute contraindication for paracentesis
d) The u/s guided dynamic approach to paracentesis is not as safe as the static approach

3. Regarding u/s probe movements, which statement is TRUE:


a) In TRANSVERSE view, moving the probe to the patients LEFT, will move the area of interest to
the RIGHT on the screen
b) In TRANSVERSE view, moving the probe to the patients LEFT, will move the area of interest to
the LEFT on the screen
c) In LONGITUDINAL view, moving the probe to the patients head, will move the area of interest to
the LEFT on the screen
d) In LONGITUDINAL view, moving the probe to the patients feet, will move the area of interest to the
RIGHT on the screen

4. When trying to identify an artery (as opposed to a vein) on ultrasound, which statement is TRUE:
a) the artery will be MORE pulsatile
b) The aorta will always be on the LEFT of the vein
c) The arterys diameter will NOT vary with respiration
d) the artery will be MORE easily compressible

5. The MOST common complication of Paracentesis is?


a) Death
b) Infection
c) Ascitic fluid leak
d) Hemorrhage

I - 81
Ultrasound Skills and Peracentesis Quiz Contd

6. Angling the u/s probe from side to side whilst maintaining the SAME position on the skins surface is MOST
ACCURATELY called:
a) Angling
b) Sliding
c) Sweeping
d) Scanning

7. If a structure is DARKER than its surrounding structures, it would be termed:


a) Echolucent
b) Echogenic
c) Anechoic
d) Hyperechoic

8. Which of the following is an indication for Therapeutic paracentesis:


a) Relief of Abdominal pain in a an with known colon cancer
b) Relief of Shortness of breath in a man with COPD
c) Relief of Hemoperitoneum in a women with a positive pregnancy test
d) Relief of shortness of breath in an alcoholic with known ascites

9. When performing a paracentesis, which structure is most commonly injured when a major hemorrhage
occurs:
a) Inferior epigastric artery
b) Femoral vein
c) Aorta
d) Inferior vena cava

10. What is the BEST way to ensure that the needle has entered the VEIN whilst performing ultrasound guided
vascular aspiration:
a) Seeing & feeling blood flow into the syringe held in your hand
b) Seeing a long pointy hyperechoic structure inside a circular structure in transverse view whilst
holding the probe in a static position above the needle insertion point
c) Seeing the tip of a long pointy hyperechoic structure inside a circular structure in transverse view
whilst sliding the probe parallel along the skin above the needle tip
d) Calculating and then advancing the needle to the depth needed to access the vein if entering the
skin at 45 degrees then advancing 1.4 times the depth of the vessel as demarcated by the distance
markers on the side of the ultrasound screen.

I - 82

S-ar putea să vă placă și